MBE Wrong Answers - General

Pataasin ang iyong marka sa homework at exams ngayon gamit ang Quizwiz!

QID: 7102 A state legislature recently enacted a statute requiring that the state department of education establish "school-mobiles" huge buses with classrooms inside, which could be driven to any location at which children were present to provide instructions, by a teacher who accompanied the bus driver, in various subjects. The state would provide all materials in the bus, such as texts and art supplies, and the teachers in charge of the school-mobiles would be department of education employees. The legislation was prompted by a report of a legislative committee which stated that numerous private schools in the state were not providing an adequate education to elementary school children, which hampered the affected children's ability to learn in subsequent, higher education and even to perform their jobs once they became working adults. Although there had never been officially sanctioned segregation in the state, patterns of housing in most school districts resulted in a large concentration of non-white students in the school districts which served the central areas of the state's major cities; most of the suburban students were white. The rapid increase in enrollments in and numbers of private schools in the state coincided with a period when many suburban school districts joined with inner-city districts in voluntary busing plans designed to bring enrollments at all schools more in balance with the percentage of people of various races or ethnicity in the state population at large. The Metropolis Academy of Upwardly Mobile Youngsters (MAUMY) is a private organization that operates an elementary school in a Metropolitan suburb. The bylaws and charter of MAUMY, specify that no non-white students may be admitted to the school operated by the organization. MAUMY has been designated by the state department of education as requiring a visit by a school-mobile once a week. If a plaintiff has standing to sue the state department of education to enjoin visits by any schoolmobile to the MAUMY's elementary school, which is the most likely reason why the court would issue an injunction? (A) The creation and operation of school-mobiles fulfills no legitimate governmental function with regard to private schools. (B) The creation and operation of school-mobiles furthers segregation when the buses are sent to the MAUMY school. (C) State aid to private schools is unconstitutional. (D) Private discrimination is forbidden by the Constitution.

(B). A court will enjoin government action that encourages or facilitates discrimination in violation of the Fourteenth Amendment Equal Protection Clause. Here, the school-mobile visits are "state action" directly benefiting a private school that discriminates against racial minorities. State-sponsored discrimination on the basis of race must be necessary to advance a compelling government interest to justify the state's actively assisting intentional racial discrimination. The Constitution allows purely private acts of discrimination. The constitutional guarantees of equal protection of the laws only protect against government infringement of those rights, not private acts of discrimination. Congress and state legislatures have the power to punish private acts of discrimination, but these are legislative acts, not Constitutional restrictions.

QID: 6669 A brother and sister grew up in the same modest home with similar educations, similar opportunities, and the affection of their parents. However, as adults, their lives greatly differed. Although hardworking, the brother's family was poor and struggled with the basic necessities. In contrast, the sister became a wealthy industrialist. Still, the brother and sister were close, so the sister eventually asked the brother and his family to live in a guest house on her estate. After 25 years, the sister became concerned for her brother's future on the estate. Much of the sister's fortune had already passed to her grown children, and she worried that if her children took over, they would evict her brother. So, the sister had her lawyer draft a document containing the following agreement: "I hereby agree to purchase for my brother, in his name, the house specified in addendum number one, said house being located downtown and not on my estate. In exchange, my brother agrees to make no claims against my estate after my death or incapacitation." Both the brother and the sister signed and dated the document. Thereafter, the sister's children seized control of the family empire, confined the sister to a sanatorium, and (as the sister's conservator) disavowed the agreement with the brother. The brother sued his sister's children for breach of contract. The children asserted that the agreement was not enforceable because the brother's promise was not supported by consideration. In most jurisdictions would the brother's agreement to make no claims against his sister's estate be regarded as sufficient consideration for the sister's promise? (A) No, if in fact the brother did not have a valid claim against his sister;s estate. (B) No, if the brother never intended to sue his sister's estate. (C) Yes, if the brother reasonably believed that he had a valid claim against his sister's estate. (D) Yes, if the brother regardless of whether he believed he had a claim against his sister's estate, did in fact refrain from making a claim against the estate.

(C). A promise is unenforceable unless supported by consideration. This is the bargained-for exchange that is so essential to modern contract law. There must be reciprocal inducement to enter the contract. Where a promise is made in exchange for another promise, the requirement is usually met. Sometimes where a promise is illusory or based on a pre-existing duty, the contract will fail. Here, if the brother believed that he had a reasonable claim against his sister's estate in the event of her death or incapacitation, and he promised to give up that claim in exchange for her promise to buy him a house, the requirements of bargained-for exchange have been met, and this is the correct answer choice. The validity of the claim itself does not determine whether the promise of its abrogation is sufficient consideration. If the brother reasonably believed that he had a valid claim against his sister's estate upon her death or incapacitation, then promising to forgo such a claim is sufficient consideration. The court, examining the document at issue, would not first go to the great length of determining whether or not the brother would have won a claim against the estate before deciding if his promise constituted sufficient consideration.

QID: 94954 A man died and was survived by his daughter. The man's will bequeathed all of his assets to his nurse. The nurse only began working for the man for the last two months of his life. The man was a citizen of State A, while his daughter was a citizen of State B. The nurse was a citizen of a foreign country, and in the United States on a work visa. The man's estate consisted of a house that was worth $176,000. The man's daughter sued the man's estate in federal court, arguing that the will should be invalidated. Does the federal court have jurisdiction over the suit? (A) Yes, because the daughter is a citizen of State B. (B) Yes, because the nurse is a foreign citizen. (C) No, because the daughter is trying to invalidate a will. (D) No, because the man's assets are only real property.

(C). Under the probate exception, federal courts typically will not exercise jurisdiction over probate matter such as the validity of a will and the administration of an estate even when the requirements for diversity jurisdiction are met. The exception is intended to preserve state control over wills and estates. Here, the daughter is trying to invalidate the man's will, so the probate exception would apply.

QID: 80879 A college freshman signed a valid written contract for the rental of a storage unit where he could safely store ski equipment. The rental agreement went into effect on September 1 and continued through August 30 of the following year. A monthly payment of $50 was due on the first day of each month and the contract contained a clause prohibiting assignments without the express permission of the owner. The student made timely payments for September, October, November, and December, but while home for the holiday break he decided to transfer to another college for the spring semester. Without informing the owner of the storage facility, he gave the key to the unit to his roommate, who wanted to take over the monthly payments and use the unit for himself. The roommate made the required payments in January, February, March, and April, but poor grades caused him to be dismissed and no further payments were made by either student to the facility. Which of the following accurately describes the legal remedy available to the owner of the storage facility? (A) The owner may sue the original render only and receive damages in the amount of $400. (B) The owner may sue the original renter only and receive damages in the amount of $200. (C) The owner may sue the roommate only and receive damages in the amount of $200. (D) The owner may sue either the original renter or the roommate and receive damages in the amount of $200.

(D). This answer is correct because contract rights are generally assignable even when the contract contains a prohibition clause. An anti-assignment clause does not completely bar assignment, but rather, an assignment made in violation of the anti-assignment clause is still valid, it simply means that the party has a cause of action against the other party for damages for violating the provision. This is the case because if the assignee successfully completes the promisor's contractual obligations, no harm results and the promisee suffers only nominal damage, if any. However, a promisor who assigns his performance obligations is not relieved of liability. He remains liable to the promisee along with his assignee. If the assignee breaches the contract, the promisee may sue either the assignee, his assignor, or both, but he is allowed only one recovery. Therefore, the owner may sue both the student and his roommate for the four months of unpaid rent, but he may only recover a total of $200.

QID: 94967 A supermarket employee filed an employment discrimination suit in federal court against the supermarket she worked for. Her suit claimed that her manager had asked her out on a date and then gave a promotion she deserved to someone else when she spurned his advances. The supermarket responded that the manager was justified in not giving the employee the promotion because she was more interested in dating the customers than she was in working. During discovery, the supermarket sought to discover detailed information about the employee's sexual history, including the names of every person she had sex with in the past 10 years. The employee refused to answer deposition questions related to this subject. and the supermarket brought a motion to compele. The court denied the motion to compel and granted the employee's cross-motion for a protective order. Was the court correct in granting the protective order? (A) Yes, because of the unreasonable intrusion on the employee's privacy. (B) Yes, because information of this nature is never appropriate for discovery. (C) No, because the information is evidence of habit. (D) No, because the information is evidence of motive.

(A) A court may enter a protective order limiting a discovery request even if the evidence is relevant.. Here, the informant's relevance is likely outweighed by the extremely personal nature of the information and its potential effect on other litigants suing for sexual harassment. There is no blanket rule stating this kind of information is never appropriate. If this were a situation where the relevance outweighed the privacy intrusion, the court would likely compel its production. "Never" is rarely a good choice on a MBE question.

QID: 7968 At 10 am, an owner sold his oil field to an investor for $800,000. The owner gave the investor a general warranty deed. The investor immediately sent the deed to her attorny with instructions to record it the following morning. On her way out of her meeting ith the owner, the investor passed a famous financier in the hall. Unbeknownst to the investor, the financier was on his way in to meet with the owner as well. Noticing the investor, the financier was curious as to what business she could have with the owner, but he decided not to let it trouble him. At 11 AM, the owner gave the financier a general warranty deed to the same oil field he had just sold to the investor, in exchange for $1 million. The financier immediately signed the property over to his wife, in order to make up for having forgotten their anniversary. The financier's wife immediately recorded the deed. The property is located in a race-notice jurisdiction. In an action by the financier's wife to quiet title to the property, how will the court rule, and why? (A) The investor owns the proeprty, and the financier's wife is entitled to damages of $1 million from the original owner. (B) The investor owns the property, and the financier's wife is entitled to no damages. (C) The financier's wife owns the property and is entitled to damages of $1 million. (D) The financier's wife owns the property, but is not entitled to money damages.

(A). A purchaser is charged with inquiry notice of any prior claims to the property if he is aware of facts or circumstances, that would lead a reasonable person to inquire further. In this case, the financier was on notice that the investor had just met with the owner, and his suspicions were raised; as such, the financier was obligated to make further inquiries regarding prior claims to the property. Furthermore, because the financier never had title to the property, he could not pass title to his wife. As such, the investor, not the financier's wife, owns the property. Nevertheless, the financier's wife is entitled to damages, because the owner breached the covenant of quiet enjoyment when he purported to sell the property to the financier. The covenant of quiet enjoyment runs with the property, permitting remote grantees to sue for its breach.

QID: 105062 Developer seeks to build a discount shopping center on a parcel it owns next to the highway in City. To do so, City must grant Developer a zoning variance, because the parcel is zoned for single-family housing. A city councilmember is concerned about the lack of open space for children to play in City. She proposes that City grant the zoning variance and allow Developer to build the shopping center, but that, in return Developer scale down the shopping center appropriately so that 10% of the parcel is devoted to a park that includes a playground and a picnic shelter. What would be the city councilmember's best argument in favor of the proposal? (A) That the proposed zoning change creates the need for the park. (B) That governments need not compensate landowners for incursions on their property rights that are for the common good. (C) That the City's propoasal advances a compelling state interest in having places for chidlren to play. (D) That the City is taking the land for public use.

(A). An exaction is a fee or land dedication that a property owner must give in exchange for developing property which offsets the effect of the development. So, for instance, if the development is on an open lot in which children often play, the requirement of creating a park for children in part of the space is such a nonmonetary offset.

QID: 12624 A farmer agreed to sell, and a tomato and vegetable dealer agreed to buy, all of the farmer's growing tomato crop for $1000, to be paid for by the dealer when delivered. The farmer then bought a bull from a rancher and, in writing, assigned to the rancher, in payment for the bull, $750 of the amount to be paid by the dealer for the tomato crop, which was not yet grown. The farmer then bought machinery from the dealer for $500 prior to the time the crop was delivered to the dealer. In spite of several demands, the farmer has refused to pay the dealer the $500. After the crop was delivered, the rancher demanded $750 from the dealer. Can the dealer set off the $500 the farmer owes him before paying the balance to the rancher? (A) No, unless the contract for the machiery, was made before the rancher gave the dealer notice of the assignment. (B) No, under any circumstances. (C) No, unless the farmer is insolvent. (D) Yes.

(A). Because the qualifying adverb "unless" is used, this choice is better understood if it is read that the dealer can set off the amount owed by the farmer if and only if he had no knowledge of the assignment at the time he sold the machinery. That correctly states the law. Once he is notified of a valid assignment, he loses the right to set off the amount due him from the assignor. If he knew of the assignment at the time he sold the farmer the machinery, he would only be able to set off $250, not the full $500 that he was owed.

QID: 108867 State X's Department of Licensing promulgated a new regulation that requires any person applying for a daycare operator's license to pass an English fluency exam. An applicant was unable to pass the English fluency exam because the applicant was an immigrant from a non-English speaking country. The applicant challenged the denial of her license application on equal protection grounds. The court applied strict scrutiny to the regulation and ruled that the Department's regulation was unconstitutional. Did the court correctly apply strict scrutiny review? (A) No, becasue the applicant did not demonstrate discriminatory intent. (B) No, because the law is facially neutral. (C) Yes, because the regulation creates a suspect classification. (D) Yes, because the regulation has a discriminatory effect based on national origin and alienage.

(A). Discriminatory intent, meaning purposeful discrimination, must be shown in order ot trigger strict or intermediate scrutiny. Where a law appears neutral on its fact, but in its application has a disproportionate effect on a particular class of persons, strict or intermediate scrutiny will apply only if the court finds that a discriminatory purpose exists. Here, the regulation appears neutral on its face because it applies to everyong. Although it likely has a disproportionate effect on aliens, there will likely be some citizens who are also affected. Therefore, there would have to be additional facts establishing that there is a discriminatory purpose to the law, and this question does not include additional facts to establish a discriminatory purpose. Therefore, the court should not have applied strict scrutiny.

QID: 95047 A driver, who was a State C citizen, was injured when the brakes of his car did not work while he was driving. The brakes were manufactured by an auto parts manufacturer which was a State A corporation with its principal place of busienss and headquarters located in State B. The driver brought a cause of action against the auto parts manufacturer in federal District Court in State B. Service of process was sent to the residence of the president of the auto parts manufacturer in federal District Court in State B. Service of process was sent to the residence of the president of the auto parts manufacturer. Service was accepted by the president's 19 year old son. The son forgot to leave his father the papers and the president of the auto parts manufacturer did not learn of the lawsuit until one month later. The auto parts manufacturer's first motion was under Rule 12(B)(5), Insufficient Service of Process. The court ruled that the auto parts manufacturer waived its right to complain about service of process. Was the court's action proper? (A) No, because service of process must be made to an officer or agent of the corporation. (B) Yes, because the motion was not made within 21 days of service of process. (C) Yes, because service was accepted by a person at the president of the auto manufacturer's residence who was more than 18 years of age. (D) Yes, because the auto parts manufacturer waived its right by appearing in court.

(A). FRCP requires that service of process on a corporation be accomplished by delivering a copy of the summons and complaint to an officer, a managing or general agent, or any other agent authorized by appointment or by law to receive service of process. There is no reason to believe that the son was an agent by appointment or law, and as such, service was ineffective. Certain defenses are deemed waived if there is a failure to raise an objection in an answer or pre answer motion to dismiss. However, here, service was never achieved, and the auto parts manufacturer did not have actual notice of the claim. Since service was not proper it does not matter that the 21 day period to file an answer had passed. Thus, the auto parts manufacturer cannot be deemed to have waived its challenge to service of process.

QID: 7580 An employee worked for the city for more than three years. During that time, the employee received several promotions within the public works department. However, the city terminated him when the public works director learned he had been convicted of selling marijuana. Thereafter, the employee brought suit in federal court under 42 U.S.C. 1983. The employee alleged that he was discharged without notice of reasons and without a hearing in violation of his constitutional rights to due process. The employee alleged that this action implemented and executed a policy, statement, regulation, and custom of the city. During the trial, the employee attempted to call the city attorney as an adverse witness. The city attorney objected and raised the attorney-client privilege. The court's ruling on the objection will be based on (A) federal common law (B) law of the forum state. (C) either the federal common law or state law, depending on which is more likely to find the subject evidence admissible. (D) the court's determination whether to apply federal common law or the state law of the forum state.

(A). Federal Rule of Evidence 501 states that "...the privilege of a witness...shall be governed by the principles of the common law as interpreted by the courts of the United States in the light of reason and experience." In other words, on matters governed by federal substantive law--and this will generally be true in criminal cases brought by the United States and private federal question cases--federal courts are to apply and develop a privilege. Because this question deals with a federal quesiton, the issues should be resolved udner federal privilege law.

QID: 80945 Plaintiff is suing Defendant for fraudulent misrepresentation concerning a parcel of real estate. During the Plaintiff's case-in-chief she offers a witness who testifies that he is a long-time acquaintance of the Defendant and that he has observed the Defendant on several occasions in the past make false statements ot people interested in buying real estate from him. The defendant objects. Is the testimony admissible? (A) No, as improper impeachment. (B) No, as improper character evidence. (C) Yes, as substantive evidence. (D) Yes, for impeachment only.

(C). In a civil suit involving a claim of fraud, the character of the defendant is in issue. Accordingly, either party may offer evidence of the defendants character for deceitfulness/honesty. This evidence may take the form of opinion, reputation or specific acts of conduct. Here, the plaintiff offers evidence of specific acts of conduct by the defendant showing his character for dishonesty. It is therefore, admissible character evidence All admissible character evidence is admissible substantively.

QID: 94989 A doctor was convicted of murder after the jury found that the victim had been having an affair with the doctor's wife and that he had prescribed medication to the victim that caused a lethal allergic reaction. The jury believed the doctor knew about the affair prior to the victim's death. The victim's family sued the hospital that the doctor was an employee of under the theory of respondeat superior. The hospital responded that the hospital would only be responsible under respondeat superior for negligence, not intentional crimes like murder. The hospital moved for declaratory judgment, offering the doctor's criminal conviction as proof that the death was intentional and not the responsibility of the hospital. How should the court rule? (A) Grant the motion, because the issue of intent was decided in the criminal proceeding. (B) Grant the motion, because a declaratory judgment will terminate the controversy. (C) Deny the motion, because a criminal conviction cannot be used in a civil proceeding. (D) Deny the motion because a declaratory judgment would be inappropriate.

(A). If the doctor has been convicted of a crime that has an element common to a subsequent civil proceeding, the conviction may have issue preclusive effect in the civil case, provided the issue was fully and fairly litigated and was necessarily decided. The issue of intent would have been essential to the murder conviction, so the criminal proceeding's decision would have preclusive effect on the civil matter. Consequently, the issue of intent would not be relitigated.

QID: 80882 A whole foods supermarket entered into a valid written contract with a local farmer who specialized in the growing of organic vegetables. The term of the contract specified that the farmer would be the exclusive supplier of vegetables for the store for a five-year period in exchange for 50% of the profits generated by their sales. The vegetables proved to be immensely popular with consumers and the store experienced a huge increase in profits as a result. Three years into the contract, the farmer informed the store owner that he had entered into an agreement of sale with an agricultural co-op for the purchase of his farm. None of the co-op's employees had any experience with organic farming. If the store files an action to prevent the same until the expiration of the contract, which result is most likely? (A) The store will win, because it is highly likely that their profits will be significantly diminished. 9B) The store will win, because there is no mention of a delegation clause in the contract. (C) The store will lose because contract duties are generally assignable. (D) The store will lose, because the co-op can continue to supply vegetables for the duration of the contract.

(A). If the sale goes through, the store can argue that there is a high probability of significant loss in profits, particularly because the co-op has no experience with organic vegetable production. Organic farming is a highly specialized skill, one that formed the basis of the contract between the store and the farmer. The farmer's attempt to sell his farm amounts to a delegation of a highly personal skill, which is not allowable when a personal service is the subject of a contract. The question is based on the delegation of duties, not the assignment of rights

QID: 8456 A rental company was burglarized one night. The night guard made a written description to police of someone he saw in the yard the night of the burglary. The night guard was killed in an accident a few weeks later. The defendant was later arrested and charged with the crime. At the trial, the state wants to offer into evidence the night guard's description. The description (A) should not be admitted because it is hearsay not within any exception. (B) should not be admitted because it is not the opinion of an expert (C) should be admitted because it is the identification made by the night guard of the person who committed the crime. (D) should be admitted because it is a past recollection recorded.

(A). In Crawford v. Washington, the SUpreme Court statd that, in accordance with the intent of the Sixth Amendment Confrontation Clause, prior "testimonial" evidence is inadmissible unless (1) the declarant is unavailable and (2) the defendant had a prior opportunity to cross-examine the declarant. Actual cross-examination is not required, but the opportunity to do so, such as at a deposition is required. Because the night guard died before the defendant was arrested for the crime, the defendant did not have the opportunity to cross-examine the night guard. Therefore, the night guard's statement even though he is an unavailable witness, should be excluded.

QID: 7501 A dog owner filed a lawsuit against a veterinarian for emotional distress arising from an injury to her dog. The dog had a rare blood disease for which the vet prescribed medication. After being administered the medication, the dog developed a massive blood clot, which resulted in the emergency amputation of the dog's right hind leg. The veterinarian denied liability, claiming that nothing in the medication could have caused the massive blood clot. The do owner seeks to introduce the testimony of a witness who is the head of another veterinary clinic. The witness intends to testify that in the past 12 months, he has treated five other dogs with the same rare blood disease using the same medication and that those dogs suffered from the same kind of blood clotting in the right hind leg as the dog owner's dog. The veterinarian objected to this testimony. Should the trial judge find the testimony admissible or inadmissible? (A) Admissible, because it is relevant to a causal connection between the dog's ailment and the medication. (B) Admissible, because the witness is a qualified expert as the head of a veterinary clinic. (C) Inadmissible, because the testimony regarding the other dogs constitutes hearsay. (D) Inadmissible, because it violates the Best Evidence Rule.

(A). Relevant evidence is evidence that has any tendency to make a fact that is consequential to the litigation more probable or less probable than it would be without the evidence. A civil litigant may attempt to introduce evidence of prior similar incidents as circumstantial evidence to support the proponent's claim or defense. As a general rule, most courts will not admit evidence of prior incidents as proof of a defendant's negligence and will not admit evidence of the lack of prior incidents as proof that a defendant exercised due care. The probative value of such evidence is simply too weak when balanced against the risk of unfair prejudice and a waste of court time. However if the evidence is offered to prove (1) that a dangerous condition existed; or (2) that the defendant was aware of the dangerous condition, the evidence will be admissible on such grounds. Here, evidence that five other dogs treated with the same drug had suffered blood clots identical to the dog owner's dog tends to prove that the medication at issue caused the dog's injuries. This causal link is a necessary element of the dog owner's cause of action, and so the evidence is relevant and admissible if there is evidence that the medication caused the same injury to other dogs under circumstances materially similar to the case at hand. It is true that the fact that a witness qualifies as an expert permits him to testify in the form of opinions that would otherwise be inadmissible. Here, however, the witness would be testifying that he treated other dogs with medical conditions similar to the dog owner's dog. Thus, his testimony would not be opinion testimony, but testimony by an eyewitness. Furthermore, even if the witness is a qualified expert, that alone will not render his testimony admissible. To be admissible, expert opinion must be relevant to a material issue in the litigation.

QID: 80814 An usher at a grand old movie theater learned that the theater's manager had recently inherited a rare print of the usher's favorite movie. The usher asked the manager if he could borrow it. The manager declined, but said he would bring the print into the theater so the usher could see it. The usher had been reading up on the subject of ESP quite a bit, and mistakenly believed there was an extremely high likelihood that he could move large objects with his mind. When the manager brought the print into the theater, the usher pointed up to the enormous chandelier above the manager's head and said that if the manager did not give him the print, he would use the power of his mind to make the chandelier fall on the manager's head. The manager looked up at the massive chandelier, laughed and told the usher he was insane to think he could move the chandelier with his mind. Nevertheless, the manager handed the print over to the usher, who was relived as he never had any intention of using his self-perceived powers of ESP to hurt the manager. While the usher had intended only to keep the print temporarily and then return it to the manager, he enjoyed it so much that he decided to keep it permanently. The usher was charged with robbery. What is the defendant's best defense to the charge of robbery? (A) The manager did not fear that the chandelier would fall on him. (B) The usher did not have the ability to make the chandelier fall on the manager. (C) The usher never intended to use force against the manager. (D) The usher did not intend to deprive the manager of the print permanently at the time of the taking.

(A). Robbery consists of all the elements of larceny, plus two additional elements: 1) the taking must be form the person or in the presence of the person, and 2) the taking must be accomplished by force or violence or by intimidation of the threat of violence. If the robbery is by threat of violence, the victim must be in actual fear at the time of the taking. Here, the manager indicated he was not in fear of the usher using ESP to hurt him. Thus, while the usher may well be guilty of larceny, he would not be guilty of robbery.

QID: 95040 A movie studio wanted to purchase a script from a screenwriter for $10,000 to turn it into a movie. Before the studio could sign a contract with the screenwriter, the studio received a letter from an author who claimed that he had written the script and that the script was stolen from him. The movie studio, a citizen of State X, brought a statutory interpleader action in the appropriate U.S. District Court against the screenwriter and the author seeking a declaratory judgment as to which of them owned the script and had the ability to sign a contract. The screenwriter was a citizen of State Y, and the author was a citizen of State X. Which of the following statements is most accurate? (A) The federal court has subject matter jurisdiction over this action. (B) Nationwide service of process is unavailable. (C) Venue would be proper in State Y. (D) Only a state court is competent to hear this matter.

(A). Statutory interpleader is authorized. It allows a person holding property over $500 which may be claimed by two or more adverse claimants to interplead all possible claimants. The federal court has diversity jurisdiction as long as there two or more claimants of diverse citizenship. Here, one claimant lives in State X and one in State Y, and the disputed amount is $10,000. The citizenship of the movie studio is irrelevant. Thus, the federal court has statutory interpleader jurisdiction.

QID: 90804 A state held a referendum whether to allow casino gambling in a certain ocean side community in the state. The referendum passed handily. Based on this, certain landowners prepared to sell their properties to hotel corporations and the like. However, at a town council hearing, environmentalists testified that unrestricted construction in the shoreline area would adversely affect the retention of sane on the shoreline, which has been eroding at a rapid rate. Faced with this testimony, the town council passed an ordinance that restricted the height of buildings within 300 feet of the ocean to no more than 3 stories. A landowner sued to enjoin the enforcement of the ordinance. How should the court rule? (A) In favor of the town, because the ordinance did not constitute a taking. (B) In favor of the town, because although there was a taking, the ordinance was a valid use of police power, thus no compensation need be paid. (C) In favor of the landowner, because the ordinance constituted a taking, as it denied the landowner all reasonable economically viable use of his land. (D) In favor of the landowner, because the ordinance did not provide for compensation for the taking of land.

(A). The Fifth Amendment provides that private property shall not be taken for public use without just compensation. This prohibition applies to the states through the Fourteenth Amendment. Property can be taken by various methods: eminent domain; inverse condemnation; or police power (whereby no compensation need be paid). When a state validly regulates for health, safety, or welfare purposes under its police power, the government action is only a regulation and does not require compensation. A taking generally results where there is an actual appropriation, destruction, or permanent physical invasion of one's property. In order not to constitute a taking, a land use regulation must substantially advance a legitimate state interest and not deny the owner all reasonable economically viable use of his land. To analyze regulations that merely decrease economic valid, courts use a balancing test to determine if there is a taking. As the ordinance advanced a legitimate state interest (to prevent erosion on the beach_) and did not deny the owner all reasonable economically viable use of his land, it did not constitute a taking.

QID: 98271 A man, a citizen of State A, brought an action in the United States District Court in State X agaist a car dealership which is a state X corporation. The complaint alleged that the car that was sold by the car dealership to the man ahd a defective engine in it, which stalled on the highway causing an accident. The man requested $76,000 in damages for the injuries that he suffered because of the accident. Process was served personally on the president of the car dealership. After receiving the complaint the car dealership filed a counterclaim against the man for $28,000 alleged to be due, because the man never paid for a van that he purchased from the car dealership two months earlier. How should a court rule on a motion to strike the car dealership's counterclaim? (A) The court should grant the motion because the amount owed to the car dealership is only $28,000. (B) The court should grant the motion because the car dealership's counterclaim is compulsory. (C) The Court should deny the motion because the man has implicitly consented to personal jurisdiction by commencing an action in State X. (D) The Court should deny the motion because there is ancillary jurisdiction.

(A). The car dealership's counterclaim against the man is totally unrelated ot the occurrence upon which the latter is contending that the former is liable. The counterclaim does not arise out of the same transaction and occurrence as the originalclaim, therefore it is a permissive counterclaim and supplemental jurisdiciton will not apply. In order for the court to hear the counterclaim, it would have to involve a Federal Question or there would have to be Diversity Jurisdiction (parties of different states and amount in controversy greater than $75,000). The case does not involve a Federal Question so that will not apply. While the parties were citizens of different states, the amount in controversy was only $28,000 so there would be no Diversity Jurisdiction. Since there is no subject matter jurisdiction for the counterclaim, the motion to strike it will be granted as the U.S. District Court does not have subject-matter jurisdiction over the car dealership's counterclaim.

QID: 7703 A driver was arrested and charged with involuntary manslaughter after a head-on collision that killed the other driver. The prosecution claimed that the driver fell asleep while driving and drove into oncoming traffic. At trial, the prosecution called a witness to testify. The witness testified that she was visiting with the driver the morning of the collision when the driver told her he had not gotten any sleep the night before because of a terrible migraine headache. The driver objected to the witness's testimony. The court should rule that the witness's testimony is (A) admissible as non-hearsay. (B) admissible as a statement of mental or physical condition. (C) inadmissible as more prejudicial than probative. (D) inadmissible, because it was not against the driver's interest when he said it.

(A). The prosecution is offering the testimony of the witness as a statement made by the driver, a party-opponent. Statements of party-opponents are admissible and are treated as non-hearsay under FRE. The statement by the driver would not qualify as a statement of physical or mental condition because under FRE, when such statements are made to non-physicians, they must relate to an existing emotional, physical, or mental condition. In this instance, the driver was not stating that he was sleepy, but that he had a migraine the night before.

QID: 7985 An attorney and her daughter had been estranged for many years, in part because of their divergent political beleifs. The attorney's will specifically disinherited her daughter and devised all of the attorney's property, including her 100-acre ranch, to a local hospital. One evening, the attorney had a change of heart and obtained a pre-printed deed form, completed it so that it purported to convey the ranch to her daughter, signed it, and sent it to the county authorities, where it was duly recorded. The attorney then wrote a letter to her daughter, stating, "The ranch is now yours forever." This letter was misplaced by the postal suthorities and not delivered. When the attorney failed to ehar form her daughter, she sold the ranch to a land-inivestment corporaiton. The corporaiton immeidately recorded the deed. The next day, the postal service found and finally delivered the attorney's letter to her daughter, who immediately emailed her mother and wrote, "Keep the ranch. I will have nothing to do with you." A week later, the daughter was killed in a car accident. Her will devised all of her property to her husband. The attorney died a month later, and her will was admitted to probate. The corproaiton, hospital, and the daughter's husband all claim ownership of the ranch. The jurisdiction has a pure notice recordation statute. Which party has valid title to the property? (A) The corproation, because the daughter rejected the gift form her mother. (B) The corporation, because the attorney had no interest in the ranch when her will was admitted to probate. (C) The daughter's husband, because recordation of the deed to the daughter creates a presumption of delivery. (D) The daughter's husband, because the corporation had constructive notice of the daughter's interest.

(A). The requirement that there be a valid delivery of a deed in order for a conveyance to be effective. Delivery is not complete until the conveyance is accepted by the grantee. The law recognizes that a grantee is normally pleased to receive a conveyance of land, and most courts imply that a grantee has accepted a beneficial transfer of property; some will do so even if the grantee is unaware fo the transfer. However, a grantee is permitted to disclaim title if she does not want the property, and an express rejection of a conveyance of land prevents delivery from occurring. When the attorney made the gift of the property to her daughter and recorded it, this gift was only presumed to have been accepted, because the attorney's letter was misplaced and not delivered in a timely manner. When the daughter received the letter and emailed the attorney, she rejected the gift and frustrated the attempted conveyance. The purported deed conveying property to the daughter was thereby rendered invalid, and title remained in the attorney. When the attorney made the conveyance to the corporation, the conveyance to the daughter was only presumed. If the daughter had accepted the conveyance or, at the least, never rejected it, the corporation would not have any title to the property, because the attorney would not have had any title to convey. However, when the daughter expressly rejected her gift, title reverted back to the attorney, and at that time, it would flow automatically to the corporation through the doctrine fo after-acquired title. Once the attorney came into possession of the title that she had previously purported to convey to the corporation, the doctrine of after-acquired title would operate to provide an automatic flow-through of the property to the corporation, validating its title. As such, the corporation has valid title to the property.

QID: 98334 A restaurant owner sued a chef for trademark infringement in the United States District Court for State A. Venue was proper in that court. The chef filed a motion to transfer venue to the United States District Court for the District of State B, a district where venue would have been proper, for the convenience of the parties and witnesses and in the interest of justice. After conferring, the restaurant owner and the chef informed the Court that, if the Court decided to transfer venue, they had agreed that the case should be transferred to the United States District Court for the District of State C, a district in which all agreed venue would not have been proper. May the Court transfer the case to the United States District Court for the District of State C? (A) Yes, because all parties have agreed that the court may do so. (B) Yes, because a federal district court may transfer a case to another federal district court for the convenience of parties and witnesses and int the interest of justice. (C) No, because that court is not a court where the case might have been brought in the first instance. (D) No, because venue is proper in the district where the case was originally filed.

(A). The venue transfer statute at one time limited the court's options on transfer of venue to districts where venue would have been proper originally. The statute now, however, allows the court to transfer to any such district or to any district where all parties consent to venue.

QID: 105025 Two college buddies, now a doctor and a lawyer, purchased on orchard as joint tenants. Several years later, the doctor was offered a position at a new hospital in a different state. Due to the high cost of housing, he mortgaged his interest in the orchard. Following the doctor's mortgage, what interest does the lawyer hold (A) A one-half interest as joint tenant. (B) A one-half interest as a tenant in common with the doctor. (C) A one-half interest as a tenant in common with the mortgagee. (D) A one-half interest as a tenant in the entirety.

(A). Under the majority lien theory, a mortgagee receives a lien on the poperty while the mortgagor retains legal and equitable title and possession of the property. Becasue the doctor retains legal title to his interest in the orchard, the unities of time and title remain intact. The college buddies remained joint tenants, each holding a one-half interest.

QID: 6779 When an advice columnist for a local paper received a letter from "Terribly Teary-eyed," he knew that the author was his ex-girlfriend. His response to her letter was published in the evening edition and referred to "Terribly Teary-eyed" as a completely immature person who must not know how to have a normal relationship because of her experience as a prostitute. When the ex-girlfriend read the newspaper and the response to her letter, she was mortified. Shortly thereafter, the ex-girlfriend found out that the columnist was her ex-boyfriend. She immediately sued him and the newspaper. The columnist moved to dismiss. If the court grants the columnist's motion to dismiss, which of the following, if true, is the least helpful reason why? (A) The ex-girlfriend could not prove that the columnist acted with malice. (B) The columnist's statements were not defamatory. (C) The response to the ex-girlfriend's letter was written as an opinion. (D) The ex-girlfriend cannot show she has been negatively affected because of the publication.

(A). When suing for defamation, a plaintiff ordinarily does not need to prove that the defendant acted with malice. Malice must only be proven to defeat a qualified privilege or where the plaintiff is a public official or public figure. Here, the ex-girlfriend is a private figure. As such, she does not need to prove that the columnist acted with malice, and so her inability to do so would be the least helpful reason why the columnist's motion to dismiss was granted.

QID: 92741 A religious group believes that spreading their religious beliefs is commanded by their faith and that all members of the congregation, including children, must engage in the public spreading of their religious beliefs, as well as in fundraising. A state law provides that no child under the age of 18 may solicit funds for any religious, charitable, or political group. The religious group challenges the law on behalf of the religious beliefs of its child members. Is the law constitutional? (A) Yes, because it has a rational basis. (B) Yes, because it is supported by a compelling government interest. (C) No, because it lacks a rational basis. (D) No, because it lacks a compelling government interest.

(A). Where a neutral government regulation of generla applicability produces merely an incidental burden on religiously motivated conduct, strict scrutiny is not applied and the Free Exerise Clause cannot be violated. Rational basis applies instead.

QID: 12793 A homeowner owned a home in a small town. On the lawn in front of his house and within five feet of the public sidewalk there was a large tree. The roots of the tree caused the sidewalk to buckle severely and become dangerous. A local ordinance requires adjacent landowners to keep sidewalks in safe condition. The homeowner engaged a handyman to repair the sidewalk, leaving it to the handyman to decide how the repair should be made. The handyman dug up the sidewalk, cut back the roots of the tree and laid a new sidewalk. Two days after the homeowner had paid the handyman the agreed price of the repair, the tree fell over onto the street and damaged a parked car belonging to a neighbor. The neighbor asserted claims against the homeowner and the handyman, and both defendants admit that cutting the roots caused the tree to fall. Which theory would provide the neighbor with the best chance of recovery against the homeowner? (A) Strict liability (B) Negligence (C) Conversion (D) Respondeat superior

(B). A defendant is not liable for torts committed by someone he has engaged as an independent contractor. This is because defendant has no right to control the activity of the contractor. In three situations, contrary to the general rule set forth above, defendant may be held vicariously liable for the torts of an independent contractor. If that duty is non-delegable, then the person who hired the independent contractor as well as the independent contractor will both be held liable.

QID: 92585 A speaker on a street corner calls upon a crowd to set fire to a nearby community center where members of an unpopular minority group frequently gather. The crows appears interested in the speaker's speech, but unlikely to attack the community center. The police arrest the speaker and charge her with attempting to incite a riot. Would the speaker's conviction on these facts be constitutional? (A) Yes, because incitement is not protected byt he First Amendment. (B) Yes, because the speaker actually intended that the crows engage in violent lawbreaking. (C) No, because it was not likely that a riot was going to occur. (D) No, because the police did not wait until a riot started before arresting the speaker.

(C). In order to convict a speaker for incitement, the government must prove that lawbreaking is likely. In this case, though a crowd appeared and was interested in the speaker's speech, the crowd was unlikely to attack the community center. Because there was no likelihood of a riot here, the speaker cannot be charged with attempting to incite a riot.

QID: 80781 A bus company provides bus rides from the mainland to several towns in the outskirts of State Blue, as well as a commuter bus service to the capital city. The company's contract with the state brings in nearly $5 million per year. Even with the bus service, travel into the capital city remains difficult, and the state legislature has begun to consider revitalizing an old rail line that connects the suburbs to the capital city. The bus company operator opposes the revitalization and speaks out against it on local television, thus stoking the ire of a state senator, for whom the rail line is a pet project. The senator calls for a hearing on the awarding of the commuter bus service contract and introduces a resolution in the Senate chambers stating that the bus company won the contract illegally and resolving to revoke the operator's license and renegotiate the contract for the state. The state licensing board then examines the bus company's operations and finds serious safety problems. The board revokes the operator's license after he is unable to address the board's concerns adequately. If the bus company operator challenges the revocation of his license, should he prevail? (A) No, because the impairment of the public contract was reasonable. (B) No, because the operator's buses endangered public safety. (C) Yes, because the legislature's act constituted a bill of attainder. (D) Yes, becasue the oeprator was denied due process.

(B). A license is a protected property interest that cannot be taken away without procedural due process. The operator was given notice and an opportunity to be heard: the facts state that he was "unable to address the coard's concerns adequately," indicating that he did appear before the board to counter the charges. Pursuant to eh state's police power, the state licensing board had the authority to revoke the operator's commercial busing license if the operation of the buses endangered the public.

QID: 7907 A student challenged a professor to a backgammon match; the loser to pay $10,000 to the winner. They played the match and the student won. The professor said she did not have the $10,000 in cash at that time, but that she would sell her beachfront property to the student for $200,000, which is $10,000 less than its current market value. "So, I'll pay you $200,000, and the beachfront property is mine, right?" said the student. "Agreed," said the professor, "we can close next week." The enxt day, the student received a note form the professor stating, "I regret ahat I cannot go through with our agreement to sell my beachfront property to you. Although the $200,000 purchase price we agreed to is a fair price, I just can't part with the property after all." The note was signed by the professor. The next week, when the student attempted to give $200,000 in cash to the professor, the professor refused to accept the money or to execute a deed conveying the beachfront property to the student. The student immediately commenced an appropriate action to compel the professor to accept the money and convey the property. The court should award judgment to which party? (A) The student, because he won the bet. (B) The student, because he had an enforceable purchase contract with the professor. (C) The professor, because of the Statute of Frauds. (D) The professor, because of a public policy defense.

(B). After the backgammon match, the student and the professor entered into an oral agreement for the purchase of the professor's beachfront property. Key to the student's success in this case is the written documentation of the agreement in the note from the professor. The written documentation contained a signature by the party to be charged, a description of the parties, a description of the property, and the price. This writing is sufficient to satisfy the Statute of Frauds. By trying to get out fo the agreement with this writing, the professor proved the existence of the agreement.

QID: 12656 A telemarketer and a hairstylist owned their house as joint tenants. After a heated argument, the hairstylist decided to leave the telemarketer and moved out of town. On Monday, the hairstylist signed a contract to convey her itnerest in the house to a personal trainer. On Tuesday, the hairstylist died in a car accident before the transfer of title had been made. The hairstylist's sister inherited the entirety of the hairstylist's estate. What is the state of the title to the house? (A) The telemarketer owns the hous as the surviving joint tenant. (B) The telemarketer and the hairstylist's sister own it as tenants in common (C) The telemarketer, the hairstylist's sister, and the personal trainer own it as tenants in common. (D) The telemarketer and the personal trainer own it as tenants in common.

(B). Although joint tenancies are marked by their rights of survivorship (when one joint tenant dies, the surviving tenant owns the whole of the property), that right can be terminated. Joint tenants can sever the tenancy, turning the tenancy into a tenancy in common. Here, the hairstylist signed a contract to convey her interest in the house to the personal trainer. In most states, a contract to convey will sever a joint tenancy even before title is transferred. While equitable title has transferred to the personal trainer, legal title is still in the hairstylist's name, and would therefore pass to the sister. The sister would be obligated to fulfill the contract and convey legal title to the personal trainer at closing. Until this occurs, however, the telemarketer and hairstylist's sister own the house as tenants in common.

QID: 12968 A man goes to his local grocery store that he visits frequently. When in the store he realizes he needs to hire a painter to paint his bedroom. The man asks a fellow shopper whether he knows of any good painters and the shopper tells him that he believes a particular painter is very competent and always on time. In fact, the shopper knows that the painter he is recommending is not competent and always late. The shopper only recommended the painter because the painter works for a painting company that is partially owned by the shopper. The man hires the painter and the painter is late and does a horrible job. The man then sues the shopper for intentional misrepresentaiton. Which of the following is the best answer? (A) The man will prevail because the man suffered damages. (B) The man will prevail because the man's reliance on the shopper was justified. (C) The man will not prevail because the shopper shared an opinion. (D) The man will not prevail because the man's reliance was not justified since the shopper was a stranger.

(B). An intentional misrepresentation by a defendant, made with scienter, which is material and justifiably relied upon by the plaintiff and which causes damages to the plaintiff, is actionable. If the defendant has an interest that he fails to disclose to the plaintiff, it is justifiable for the plaintiff to rely on the defendant's expression of opinion on the subject of that interest. In this case, the shopper made a false affirmative statement that he knew to be false and yet failed to disclose to the man that he owned part of the painting company. Thus, the man's reliance on the shopper was justified and the man will prevail in his suit.

QID: 6598 A young man had been helping his father run the father's farm for several years. The son wished to move out of town, but his father's health had been declining to the extent that he was unable to take care of himself anymore. The farm was located south of town, in an area into which the town might be expanding in the next few years. The son was aware of the possible expansion, but his father was not. The son spoke to his father and told him of his plan to move out of town. Worried about his father's failing health, the son urged his father to sell the farm to a neighbor for $150,000. Unbeknownst to the father, his son and the neighbor were actually planning to hold onto the land until the town expanded south, at which time they planned to subdivide it and make a large profit in the real estate market. The father agreed to sell the farm to the neighbor and signed the papers for a transfer, even though the sales price was below the actual market value. The father later discovered that he could have sold the farm for at least $300,000. He now seeks to void the contract. Which of the following is the father's strongest argument? (A) The contract is voidable by reason of misrepresentation. (B) The contract is voidable by reason of undue influence. (C) The contract is voidable by reason of unconscionably. (D) The contract is voidable by reason of mistake.

(B). Any contract that is formed because of undue influence can be voided. In determining whether a contract was formed as a result of undue influence, two key factors are unfair persuasion and a special relationship, such as that between parent and child. Here, the father trusted his son to make good decisions for him, and there is no evidence that he placed less than full confidence in his son to act in his own best interests. As the son tried to unfairly persuade his father to sell the farm so that he (the son) would benefit thereby, the father's strongest argument for voiding the contract is that it was formed as a result of his son's undue influence. A contract will be voided for a unilateral mistake only if it is a material one and the non-[arty knew or should have known of the other party's mistake. Here, while the father may have been mistaken about an assumption underlying the contract (i.e. the value of his land), the contract will not be voided unless he can show that the son knew or should have known of his mistake. Therefore, a claim of mistake is not the strongest argument for voiding the contract. Any contract that is formed because of misrepresentation or fraud can be voided. If a party agrees to a contract based on a material misrepresentation, the contract is enforceable. In this case, to show that the contract was the result of material misrepresentation, the father would need to show that the son knew or should have known that he was making a false representation of a material fact to the father and that the father relied on the misrepresentation. As it would be difficult based on the above facts for the father to prove that the son was knowingly misrepresenting material facts regarding the value of the farm, this is not the strongest argument for voiding the contract. Generally, the elements needed to arrive at a finding of unconscionability are unequal bargaining power and a one-sided contract. Here, the only potentially unconscionable term is the lower-than-market-value price paid for the farm. Given the fluctuating value of real estate and the fact that the father did not object to this price at the time, it is unlikely that he would succeed in arguing that the contract should be voided as unconscionable.

QID: 6747 A 10-year old girl was spending the summer with her family at a mountain cabin located on a lake. One afternoon, the girl's parents went to the nearby town to buy supplies, leaving the girl alone at the cabin. As soon as her parents left, the girl ran to her parents' room and took er father's toy motorboat. The firl had frequently watched her father place the toy boar in the lake and operate it with the handheld control panel and wanted to attempt to operate the boat herself, but her father had told her it was too hard for a child to properly operate the controls The firl decided this would be the day she would operate the toy boat. She ran down to the dock and placed the toy boat in the water. Using the handheld control panel, she started the toy boat's motor and engaged the throttle. The toy boar raced around the lake as the girl steered it, and she kept increasing the speed of rht boar until it was going so fast that she was having difficulty controlling it. Suddenly, she noticed that the boat was heading directly for two swimmers, a young couple. The girl tried to steer the boat away from them, but because of the speed of the boat and her inexperience in operating it, she was unable to do so. The toy boat hit one of the swimmers, causing a deep cut i his left arm. He reacted by swinging around to see what had hit him. In doing so, his right elbow hit his girlfriend in the head, causing a deep bruise. If the girlfriend brings suit against the girl, will she recover? (A) Yes, because the girl was engaged in an adult activity. (B) Yes, because the girl did not behave as a reasonably prudent child of the same age, knowledge, intelligence, and experience under the circumstances. (C) No, because her harm was caused by her boyfriend. (D) No, because a 10-year-old child is presumed incapable of the state of mind necessary for negligence.

(B). As a 10 year old child, the girl's conduct would be judged against the standard of what a reasonable child of the same age, knowledge, intelligence, and experience would have done under the circumstances. However, if the girl is engaged in an adult activity, she would be held to the tougher objective standard of what a reasonable adult person would have done under the circumstances, with no allowance for her limited maturity. Because operating a toy boat would likely not be considered an adult activity, this would be the applicable standard.

QID: 90780 A city originally had only one adult bookstore, located in an isolated part of town. The opening of this adult bookstore had no appreciable effect on the crime rate in the area. Two years later, a second adult bookstore opened on the same street. After the opening of the second bookstore, crime in the area increased by 25%. Soon after, an entrepreneur opened a strip club on the same street as the two adult bookstores. He applied for a liquor license for the club, but the state-sanctioned city liquor board denied the license without a hearing. Raising a freedom of expression argument, the entrepreneur brought an action for a writ of mandamus to compel the issuance of the liquor license to the strip club. Assuming jurisdiction and standing, how should the court rule? (A) Writ denied, because the city could deny the liquor license to the strip club based on the increase in crime in the area after the opening of the second adult bookstore. (B) Writ denied, because the city had a substantial governmental interest, and alternative channels of communication were left open. (C) Writ granted, because based on local community standards, nude dancing establishments do not appeal to the prurient interest. (D) Writ granted, because strip clubs do not depict sexual conduct in a patently offensive way, based on state law standards

(B). Certain categories of speech, including sexual speech, receive lower levels of protection from the Constitution. The regulation must serve a substantial government interest and leave open alternative channels of communication. Bans on public nudity, including nudity at nude dancing establishments, have been upheld for reasons of protecting societal order and morality and protecting against the possible secondary effects of nude dancing establishments, such as prostitution and other criminal activity. Here, the denial of a liquor license to the strip club does not ban nude dancing altogether (after all, the club can operate without the liquor license), and the city has a substantial government interest in keeping crime in check. Therefore, the writ will be denied. Note that the 21st Amendment permits the state to regulate liquor establishments, but the state may not be used as a constitutional basis to override the protections afforded by the First Amendment.

QID: 8408 In order to preserve and manage the number of fish in state offshore waters, a state legislature passed a law that required all persons who wished to fish in the state's territorial waters to obtain a fishing license. A resident commercial fishing license cost $500, a nonresident commercial fishing license cost $5000, a resident recreational fishing license cost $5, and a nonresident recreational fishing license cost $25. Several nonresident commercial and recreational fishermen filed suit to enjoin the operation of the fishing license law, contending that the law violated the Privileges and Immunities Clause of Article IV, Section 2 of the United States Constitution. Assuming proper standing, how should the court rule? (A) In favor of all plaintiffs. (B) In favor of the nonresident commercial fishermen, but against the nonresident recreational fishermen. (C) In favor of the nonresident recreational fishermen, but against the nonresident commercial fishermen. (D) Against all plaintiffs.

(B). Discrimination against nonresidents in regards to essential economic rights or liberties triggers Article IV Privilege and Immunities Clause protection.The clause protects nonresident commercial users but not nonresident recreational use because states have the right to police recreational (noncommercial) activities.

QID: 92581 After a series of car accidents on a busy stretch of urban highway, a city passes an ordinance that bans any commercial billboards along the stretch of highway. Under the ordinance, political and charitable advertisements are still permitted. A company that owns a large proportion of the existing billboards on the highway challenges the law under the First Amendment. Is the signage ordinance constitutional? (A) Yes, because commercial billboards are not protected by the First Amendment. (B) Yes, because a city may ban all commercial billboards as long as it leaves other kids of billboards available. (C) No, because the restriction is overbroad. (D) No, because the restriction violates the commercial speech doctrine.

(B). It is within a state or city's police power to prohibit billboards carrying commercial advertising under a highway safety rationale, as long as the city does not discriminate between different kinds of commercial messages on the basis of viewpoint. The First Amendment protects commercial advertising in all of its forms, including billboards. Because the city has the power to ban all commercial billboards, the regulation of all commercial billboards cannot be overbroad. The commercial speech doctrine does protect billboards, but it does not prevent a city from banning all commercial billboards under a highway safety rationale.

QID: 7586 A man sued a woman for injuries received in an automobile collision. At trial, the man called a doctor who proposed to testify that two weeks after the accident, the woman checked in to a rehabilitation for alcohol and substance abuse. Is this evidence admissible? (A) Yes, because it is relevant to establish the woman's negligence. (B) Yes, because it was a statement by a party opponent. (C) No, because its probative value is outweighed by public policy considerations. (D) No, because it is hearsay not within an exception.

(C). In this subtle questions, the woman's actions are to be construed to constitute a subsequent remedial measure. If a defendant takes action after injuring the plaintiff to reduce the chance of further injury to others, the plaintiff may not introduce those actions as circumstantial evidence of fault. This is a public policy exception to encourage remedial actions.

QID: 108859 A woman decided to give birth to her child at her home without medical assistance. Although the birth was difficult, the woman refused to seek medical aid or go to a hospital. The child died the next day from complications. An autopsy revealed that with proper medical intervention, the child would have survived. The state charged the woman with involuntary homicide for refusing to seek medical treatment for her child. The woman moved to dismiss the charges, arguing that the charges violate equal protection because she is being treated differently than the women who choose to give birth in a hospital. Will the court grant the woman's motion to dismiss? (A) No, because the state has a compelling government interest in ensuring children receive appropriate medical care. (B) No, because all women are required to provide appropriate medical care to their children, regardless of where they give birth. (C) Yes, because the charges are not substantially related to an important government interest. (D) Yes, because women have the right to choose where to give birth.

(B). The Fourteenth Amendment provides that no state shall "deny to any person within its jurisdiction the equal protection of the laws." Equal protection review is triggered where persons similarly situated are treated differently. Here, the woman is not being treated differently because all women are required to seek appropriate medical care for their children. It simply happens that women who give birth in a hospital already have provided that medical care. Therefore, no equal protection review is triggered.

QID: 98305 A plaintiff sued a defendant corporation in federal court in State A for a tort. The plaintiff had the defendant corporation duly served with process in State B, the only state where the defendant corporation was incorporated and where the corporation had its principal place of business. The corporation's tortious actions all occurred in State B. The only connection between the corporation and State A was that the corporation knew that its actions would harm the plaintiff in State A. The corporation filed a motion to dismiss for lack of personal jurisdiction. The court denied the motion. Did the court err when it denied the motion to dismiss? (A) Yes, because the defendant corporation was not served with process in State A. (B) Yes, because the defendant corporation did not have minimum contacts with State A. (C) No, because the defendant corporation knew that its actions would harm the plaintiff in State A. (D) No, because the plaintiff was injured in State A.

(B). The corporation was an out-of-state defendant in State A. Therefore, any assertion of jurisdiction over the corporation in State A had to comply with due process, which means that under International Shoe, the corporation must have had sufficient minimum contacts with State A. There are two prongs on the minimum contacts analysis, and neither allows the plaintiff to sue the corporation in State A. There is no "general jurisdiction" over the corporation in State A because the corporation is not incorporated there and does not have its principal place of busienss there. In the recent case of Daimler AG v. Bauman, the Supreme Court of the United States held that, absent some exceptional circumstances, general jurisdiction exists only in such states. There was no "specific" jurisdiction because the corporation's only contact with State A is that it knew it was causing harm in State A. In the recent U.S. Supreme Court case of Walden v. Fiore, the court held that this is not enough to establish personal jurisdiction.

QID: 8042 A managing agent of a commercial building entered into a lease agreement with a law firm. The agent agreed to lease two floors to the firm, and the law firm agreed to pay $300,000 in five annual payments of $60,000. The agreement also stated that payments were to be paid to the agent's successor corporation, if any, and that the agent retained the right to assign the agreement. One year into the lease, the law firm entered into a contract with an insurance company. In exchange for the firm's promise to perform legal services for the insurance company, the insurance company would perform the firm's duties under its lease. Soon thereafter, the managing agent changed the beneficiary form its successor corporation to that corporation's CEO. When the CEO received notice of this, he sent a latter of thanks to the managing agent and notes to both the law firm and the insurance company advising them of his present address. Subsequently, the agent borrowed $20,000 form a bank and assigned the lease agreement as security. Then the law firm and the insurance company rescinded the contract that provided for the insurance company's payment of the rent. The firm then failed to make an annual payment per its lease. The agent's bank, not having been paid the $30,000 that it lent to the agent, sued the law firm for $30,000. The CEO intervened, alleging that the bank did not have a right to any sum under the lease agreement. How should the court rule? (A) Grant judgment to the bank for $30,000, because consideration as given for the assignment. (B) Grant judgment to the bank for $30,000, because the managing agent reserved power to change the beneficiary and to assign the agreement. (C) Grant judgment to the law firm, because the CEO was named beneficiary before the managing agent made the assignment, so the beneficiary prevails over the assignee. (D) Grant judgment to the law firm, because the CEO is a donee beneficiary and the promisee cannot discharge any part of a donee beneficiary's right.

(B). The managing agent properly and validly assigned the right to payment to the bank. In a partial assignment, the assignor must transfer the right assigned completely, but may divide several rights due to him from the obligor and assign one or more of them to one or more assignees. Here, the lease agreement stated that the managing power had the power to change the beneficiary and to assign the policy. The assignment is viewed as an exercise of the power to assign, even though it was only a partial assignment.

QID: 81016 A juggler borrows money from his coworker. The juggler is unable to pay the money back, and years pass. The coworker eventually needs the money and contacts the juggler, who consults a lawyer. The lawyer tells the juggler that the statute of limitations on the debt has expired and that he is not legally obligated to repay the loan. The juggler nevertheless promises to do so in writing, but then changes his mind. May the coworker enforce the juggler's recent promise to repay the old loan? (A) Yes, because the past consideration is sufficient to support a present promise. (B) Yes, because it was a promise to repay a debt that, but for the statute of limiations, would still be owing. (C) No, because the recent promise to repay the money was a gratuitous promise. (D) No, becasue the recent promise to repay the money was supported only by past consideration.

(B). This answer is correct because it applies an exception to the general rule that past consideration will not support a contract. A written promise to pay a debt that is barred by the statute of limitations is one of two particular types of promises that are enforceable even if there is no new consideration for them. In this case, the juggler's promise to pay the debt despite the expiration of the statute of limitations is therefore binding.

QID: 8056 The owner of a soap company that manufactured laundry powder had not heard from one of his regular distributors for an order in nearly a year. The owner called the distributor, who told him that she did, in fact, need more laundry powder, and the parties agreed that the soap company would send three separate deliveries to meet her needs. The day after the telephone call, the soap company sent the distributor a letter that stated, "Based on our records, you are nearly our of your supply of our wonderful laundry powder products. We will, therefore, send to you three truckloads over three months at the rate of $3,000 per truck to restock your supplies and keep you supplied for several months." The letter was signed by the soap company owner and properly identified the parties. The soap company subsequently sent the first truckload of laundry powder to the distributor, and the distributor accepted the delivery. Several days later, however, the soap company owner heard that the distributor had already resold the laundry powder to merchants in her area. The soap company owner then shipped the next truckload to the distributor. When the distributor received the second truckload of laundry powder, that she was rejecting the delivery, and that she would also refuse to accept the future deliveries. The soap company filed a breach of contract action against the distributor, claiming that the distributor had no right to refuse the second and subsequent shipments of the laundry powder. The distributor asserted the Statute of Frauds as a defense. Which of the following is true regarding the viability of the distributor's Statute of Frauds defense? (A) The distributor will fail in asserting the Statute of Frauds as a defense because the soap company had partiall yperformed the ocntract at the time of the distributor's attempted repudiation. (B) The distributor will fail in asserting the Statute of Frauds as a defense, because under the circumstances, the soap company owner's letter satisfied the Statute of Frauds requirements for an agreement between merchants. (C) The distributor will succeed in asserting the Statute of Frauds as a defense, becasue the parties did not enter into a signed, written agreement regarding the purchase of the laundry powder. (D) The distributor will succeed in asserting the Statute of Frauds as a defense because the distrubotor did not accept the soap company's offer in writing.

(B). Under Article 2 of the UCC, the UCC Statute of Frauds may be satisfied when two merchants enter an oral agreement for the purchase and sale of goods and one party then sends the other a written confirmation of the agreement. In such a circumstance, the Statute is satisfied against the recipient merchant if the latter fails to object to the confirmation in a timely fashion. Under the UCC, a valid "merchant's confirmation" requires a writing that is: (1) sufficient against the sender; (2) in confirmation of the contract; (3) is sent within a reasonable time fo the making of the oral agreement; and (4) the contents of which the receiving merchant has reason to know. In this case, the soap company owner's signed letter is sufficient to satisfy the UCC Statute of Frauds requirement for a "merchant's confirmation." The writing identified both parties and contained all of the necessary contractual information and terms, including the quantity of truckloads of laundry powder goods over several months. The letter was signed by the shipping merchant, and the first truckloads of goods were subsequently accepted by the distributor buyer, who did not object to the contract within 10 days of receiving the letter. Moreover, the time period is reasonable and the distributor has reason to know the contents. As such, the letter constitutes a proper "merchant's confirmation," and the distributor will not succeed in asserting that Statute of Frauds as a defense.

QID: 12895 A grandfather was leasing a garage where he kept his cars and motorcycles. The grandfather knew that he had a good deal, because he had been renting that garage for a long time. One day, the grandfather learned that his son was going to need space for some of the son's cars. Since the grandfather wanted to move to another city, he decided to transfer the lease to his son. The grandfather signed a contract with the owner of the garage by which the contract would be transferred over to the con in one year and the son would receive the same rate. The contract also stated that the contract could not be modified after three months. Which of the following is correct/? (A) The son's rights in the garage lease vested when the contract was signed. (B) The son's rights in the garage lease will vest in three months. (C) The son's right s in the garage lease will not vest because he is a donee beneficiary. (D) The son's rights in the garage lease will not vest because he is an incidental beneficiary.

(B). Under contemporary law, a third-party beneficiary may have standing to sue the promisor for breach, even though the promise was made to the promisee and not to the third-party beneficiary. The question here is when do the beneficiary's rights vest. If an express term in a contract provides for vesting, then vesting with regard to an intended beneficiary will occur when that term is satisfied. In this case, the contract stated that the contract cannot be modified after three months, so the son's rights will vest after three months.

QID: 6742 The owner of a retail garden supply store ordered 500 terracotta flower pots from a wholesaler for a total price of $2500. Under the contract, the wholesaler is to deliver the flowerpots to the store on May 5. On May 1, the wholesaler delivers the flower pots to the store. Upon delivery, the owner of the store inspects the shipment. Although the wholesaler's catalog stated that the flower pots had drainage holes, the flower pots that were delivered did not. The store owner immediately notifies the wholesaler that the flower pots are defective. The wholesaler offers to replace the flower pots with a shipment of conforming flower pots to be delivered on May 4. The store owner refuses the wholesaler's offer and tells the wholesaler that he has decided to buy flower pots from a different wholesaler. Which of the following is the most accurate description of the parties' rights? (A) The store owner had the right to reject the flower pots, subject to the wholesaler's right to cure, but only if the wholesaler delivered conforming pots immediately upon learning of the defects. (B) The store owner had the right to reject the flower pots, subject to the wholesaler's right to cure. (C) The store owner had the right to reject the defective flower pots under the UCC's perfect tender rule. (D) The store owner was required to acept the flower pots, because they were still of mechantable quality.

(B). When a buyer has cotracted to purchase goods under a single delivery contract and the seller delivers nonconforming goods, the buyer has a right to reject the goods and either cancel the contract or sue for damages. (In the alternative, the buyer may keep the goods or a portion of the goods, and sue for damages under the contract.) However, the buyer's right to reject nonconforming goods is subject to the seller's right to cure. To cure, the seller must notify the buyer of the intention to cure and then deliver conforming goods to the buyer. Generally, the conforming goods must be delivered within the originally contracted period for performance. Thus, under the facts presented, the store owner had a right to reject the nonconforming flowerpots, but the wholesaler has a right to cure the defect. The wholesaler gave the store owner reasonable notice that it intended to cure the defect and offer to deliver confrming goods withing the original contract period.

QID: 28258 For four continuous years, an associate attorney at a private law firm has maintained the practice of writing the date of opposing counsel's discovery requests on the top right hand corner of her case file and having the opposing counsel sign next to the client's name whenever they receive a discovery packet. During one of the law firm's trials, opposing counsel proffers to the judge that the associate attorney had intentionally not provided discovery. The associate is fired that day as a result. In a lawsuit against her former employer, the fired associate testifies and is able to show the court all of her case files form the previous four years where the date and signatures appeared on the case files. She presented the case file for the most recent case and showed the court where there was no date and no signature in order to prove that the opposing counsel had lied about ever requesting discovery. Is it proper for the fired associate to present her case file in this manner? (A) Yes, these were the records of a public office. (B) Yes, it is an entry which is absent from her records. (C) No, this is inadmissible hearsay. (D) No, these are not records of vital statistics.

(B). When a person testifying is able to show that evidence is not included in memoranda, reports, records, or date compilations, which had been kept in accordance with the provisions of the business records rule, then that evidence can be used to prove the non-concurrence or non-existence of a matter. This is true of the evidence or documents being reviewed were regularly made and preserved. The evidence will prove that non-existence of a fact unless there are other circumstances that would indicate that there is a lack of trustworthiness. Although the evidence in the records would be hearsay, the hearsay is admissible in court because it falls under an exception to the hearsay rule as an absence of entry form records. Here, the associate made it a policy and practice to maintain her records in a certain manner. She kept these records in accordance with the business records rule and therefore, the lack of a date or signature by the defense attorney can be used as evidence against to show that no discovery request was ever made.

QID: 8305 A club manager and an investor decided to establish a new nightclub. The club manager donated the use of his loft, and the investor provided the initial start-up capital. The investor did not wish to be bothered with day-to-day operations, so he and the club manager entered into a written agreement limiting the investor's employment duties to that of official host at the new club. As the host, the investor would be responsible for maintaining a well-ordered club environment. The nightclub was a big success, and the club was packed with people on a nightly basis. To prevent the club manager form burning out form exhaustion due to the late club hours, the investor and the club manager verbally agreed that the investor would take a turn running the club on the first Friday of every month. On the first Friday in September, the investor noticed an argument developing at t table where a man and a woman were seated. He approached the table and asked if there was anything he could do for the nightclub guests. The man, who was seated at the table with his date, said "You can stop serving this swill." When the date told the man to calm down, the man became so angry that he hurled his cocktail glass to the floor, smashing it. The investor ordered the man to leave, and when the man refused, the investor physically forced him out of the nightclub. The man broke his leg when the investor tossed the man out onto the street. The man sues the club manager and the investor. Will the club manager be liable for the man's injury? (A) Yes, because the injury occurred on the club owner's property. (B) Yes, because the investor and the club manager entered into a joint venture. (C) No, because the investor's employment duties were limited to acting as host. (D) No, because the investor was privileged to use self-defense.

(B). When two or more individuals agree to enter into an undertaking in the performance of which they have a community of interest and mutual right of control, they are said to be engaging in a joint venture. Because such an arrangement is similar to a business partnership, each member is vicariously liable for any torts committed by the others within the scope of the enterprise. Here, the investor and the club manager jointly created the nightclub. The club manager provided the use of his loft, and the investor provided the start-up capital. That they both contributed to the business venture from which they would both benefit shows their joint undertaking. The investor and the club manager had a community of interest in the nightclub. As host, the investor was responsible for maintaining a well-ordered environment. While the investor initially attempted to limit the scope of his employment, he and the club manager subsequently agreed to add to the investor's duties by his managing the club on first Friday evenings. Because they agreed that the investor would run the club on certain occasions, his duties, risks, and liabilities also expanded. Both parties had a mutual right of control in the nightclub operations when they formed a joint venture. Because the investor was acting within the scope of his employment agreement and within the scope of the enterprise when he escorted the man from the nightclub and tossed him on the street, the club manager will be vicariously liable for any torts committed by the investor. Therefore, the club manager will be vicariously liable for the man's injuries and damages. This is the correct answer.

QID: 8254 A neighbor, along with several others, is watching a neighborhood game of football. It is a neighborhood tradition that whenever a player scores a touchdown, the player will rush at and tackle one of the fans on the sidelines. Soon, a player scores a touchdown. The player immediately looks toward his brother, who is watching the game and standing directly behind the neighbor, and the player charges at his brother. The neighbor, believing that she is about to be bowled over, swings her purse at the player, breaking his nose. The player sues the neighbor, and the neighbor asserts the privilege of self-defense. Who will prevail? (A) The neighbor, because the neighbor did not consent to being tackled. (B) The neighbor because even when mistaken, the neighbor has a right to defend herself. (C) The player, because the neighbor was mistaken as to her need to defend herself. (D) The player, because the neighbor could have retreated.

(B). Where a defendant is reasonably mistaken as to his or her need for self-defense, the privilege of self-defense still exists. Here, although the neighbor was not the actula target of the player's tackle, she reasonably believed that she was; as such, her defense will stand.

QID: 6659 A homeowner entered into a written contract by which a furniture designer/carpenter agreed to design book shelves and office area in the homeowner's new home within six months in exchange for $20,000. The designer/carpenter reported to the homeowner that the work was completed and demanded her $20,000. The homeowner inspected the shelving and discovered that the designer/carpenter had installed three cabinets instead of the four called for in the specifications accompanying and incorporated by reference into the contract. The homeowner estimated that it would cost $2,000 to install a fourth cabinet and mailed the designer/carpenter a check for $18,000 on which she had written "Payment in Full." The designer/carpenter cashed the check and then brought an appropriate action against the homeowner for the additional $2,000. For whom should the court award judgment? (A) The designer/carpenter, because she substantially performed under the contract, and the homeowner's remedy, if any, is to seek a judgment for damages on her own behalf. (B) The homeowner, because the designer/carpenter is estopped from denying that she has been paid in full. (C) The homeowner, because there has been an accord and satisfaction. (D) The homeowner, because the designer/carpenter is in material breach of contract and cannot, therefore, complain of the homeowner's failure to perform as promised.

(C). An accord is an agreement in which the obligee of a separate contract accepts a lesser performance than was called for by the separate contract. A satisfaction is the consummation of the accord--when the stated, lesser performance is rendered. The (often criticized) rule of Foakes v. Beer provides that when a debt is liquidated and undisputed, there can be no accord and satisfaction, because no consideration supports the obligee's acceptance of a lesser performance. However, as to a bona fide disputed obligation, the compromise of the competing claims provides consideration for the accord, and upon satisfaction, the separation obligation will be extinguished. Under Article 3 of the UCC, applicable to checks, a conspicuous "full payment" statement on a check for less than the disputed amount constitutes an accord; if the payee cashes the check, she is held to have agreed to accept the lesser payment in satisfaction of her contractual rights. Thus, when the designer/carpenter cashed the $18,000 check marked "Payment in Full," she was impliedly agreeing to accept $2,000 less than the contract price as a compromise of the dispute over the fourth cabinet with the homeowner. Having so agreed, the designer/carpenter cannot thereafter seek to enforce the full contract price.

QID: 105037 A mother brought a personal injury action on behalf of her son against a cab company for damages arising from a car accident in which the mother's car collided with a cab driven by one of the company's drivers. During his lunch break, the cab driver had consumed several alcoholic drinks. In her complaint, the mother alleged that the cab company had been negligent in approving the hiring of the cab driver, who was an alcoholic, and that the company reasonably should have known that the driver's alcoholism made him a serious risk to others while operating a motor vehicle. At trial, the mother called the cab driver's roommate to testify that she had lived with the driver for the six months prior to the accident and that, on three separate occasions, she had observed the driver operate a motor vehicle while in a severely intoxicated state. The cab company objected to the introduction of the roommate's testimony. Should the court admit the roommate's testimony over the cab company's objection? (A) No, because evidence of specific instances of misconduct may only be inquired into on cross-examination and may not be proven by extrinsic evidence. (B) No, because the cab driver has not testified that he haw never operated a motor vehicle while intoxicated. (C) Yes, because the cab driver's character is a material issue in the litigation. (D) Yes, because the cab driver is not a defendant in this case.

(C). Federal Rule of Evidence 405 holds that when the character of a person is a substantive issue in the litigation, evidence of his character is directly relevant and is admissible in any form: reputation, opinion, or specific instances of conduct. Here, the basis of the mother's complaint is that the cab company was negligent in hiring the driver and entrusting a cab to him while failing to discover that the driver drove motor vehicles while intoxicated. Thus, the matter of whether the driver in fact drove motor vehicles while intoxicated is a material issue of the case, and evidence that he had done so on previous occasions is admissible as directly relevant to that issue.

QID: 80732 A defendant was on trial for murder in a high profile case. During the second day of jury deliberations, a juror began telling the other jurors about a newspaper article regarding the defendant's past that the juror read the night before. The juror told the other jurors about the defendant's extensive criminal history, which was not admitted at trial but was in the newspaper article. The jury returned a unanimous guilty verdict. May a juror testify about the discussion of the newspaper article that occurred during deliberations? (A) No, because jurors are not allowed to testify about matters that occurred within the jury room. (B) No, because a juror must submit an affidavit to give a statement about the discussion of the newspaper article. (C) Yes, because this is proper testimony to determine the validity of the verdict. (D) Yes, because a juror may testify regarding any topic after the jury returns a verdict.

(C). Generally, jurors may not testify about matters occurring during deliberations, but there are exceptions to this general rule where extraneous prejudicial information was brought to the jury's attention, an outside influence was improperly brought into jury deliberations, or there was a mistake in entering the verdict on the verdict form. Because the juror improperly brought extraneous information (the newspaper article) into the jury deliberations, a juror would properly be able to testify regarding the discussion of the newspaper article to determine the validity of the verdict.

QIID: 108927 A defendant was on trial for the murder of his supervisor. The defendant wished to call a witness to testify that, the day before the death, the defendant told the witness that the defendant was about to leave that day for a week's vacation in a foreign country. Is the testimony admissible? (A) No, because it is hearsay not within any exception. (B) No, because it requires the finder of fact to make an assumption as to an element of the prosecution, namely that the defendant had an opportunity to commit the crime. (C) Yes, because it is a declaration of present mental state. (D) Yes, because it is relevant to establishing the defendant's alibi.

(C). Hearsay is an out of court statement offered for the truth of the matter asserted therein. The defendant's statement to the witness about his vacation is hearsay. Therefore, to be admissible, it must fit within an enumerated exception. FRE allows a statement of the declarant's then-existing state of mind (such as motive, intent, r plan) or emotional, sensory or physical condition (such as mental feeling, pain, or bodily health) to be admitted as an exception to the hearsay rule. The defendant is on trial for murder. In his defense, he will try to show that it could not have been him because he was somewhere else at the time of the killing. To do this, the defendant wants to introduce his previous statement to a witness that the defendant was about to leave for a vacation to a foreign country. The defendant wants to use this testimony for its truth, to prove he was somewhere else when the crime occurred. It will be admitted for its truth as it qualifies as a present mental state statement.

QID: 28266 At a criminal trial for murder, a total of 27 witnesses were presented before closing arguments. Ultimately, the defendant was convicted of the crimes alleged against her. The case went forward to appeal and the defense attorney arguing the appeal claimed that many of the witnesses were not competent to testify. Which of the following witnesses was not competent? (A) A juror who was seated on the jury panel in this same trial against the defendant testified that another juror used a cell phone to access a social media website and review commentary made by other witnesses during the trial. (B) An attorney who is employed by a prosecutor's office in another county testified to statements made by the defendant in open court at a preliminary hearing when this attorney was visiting and observing the preliminary hearing. (C) An unbiased bystander testified to observing the defendant fleeing after the commission of the crime but was not administered an oath. (D) A police officer qualified as a weapons expert testified as to the size and probable training of the person who fired the murder weapon. The officer had never met the defendant and had no personal knowledge of the defendant.

(C). People are generally considered to be competent unless they are subject to some specific exceptions. A competent witness is one who has personal knowledge of relevant matters and has declared by oath or affirmation that he or she will testify truthfully. An unbiased bystander who was able to observe the defendant is presumed competent to testify absent special circumstances rebutting that presumption. Here, there is no evidence that the bystander should be disqualified for reasons like felony convictions, atheism, age, mental incompetency, financial interest, or marital relation to the defendant. However, the bystander was never given an oath or affirmation agreeing to testify truthfully. This lack of oath automatically rendered the bystander incompetent to testify in this proceeding.

QID: 81416 Police received a tip from an informant that a ring of thieves were hoarding stolen property in the house owned by the ringleader's uncle. The ringleader frequented the uncle's house to sell the stolen goods out of the basement and could be found there every evening after 10 PM. The police obtained an arrest warrant and went to the uncle's house to arrest the ringleader the next evening. The uncle refused entry to the police, who pushed past him to find the ringleader. When the police entered the living room, they observed bricks of marijuana on the table. Police arrested the uncle and charged him with narcotics offenses. Was the marijuana seizure constitutional? (A) Yes, because the officers observed the marijuana in plain view. (B) Yes, because the officers observed the marijuana while executing a valid arrest warrant. (C) No, because the police did not have a valid search warrant for the uncle's house. (D) No, because there was no proof that the drugs belonged to the uncle.

(C). Police may not cross the threshold of a third party's resident to execute a valid arrest warrant unless the police first obtain a search warrant authorizing the entry into the residence or there are otherwise exigent circumstances that permit a warrantless entry. In this case, there was neither. Therefore, the seizure was unconstitutional.

QID: 90779 A state legislature enacted a statute that allowed those convicted of a misdemeanor to confess judgment with a surety in the amount of the fine and costs, and then to agree with said surety, in consideration of the surety's payment of the confessed judgment, to reimburse him by working for him upon terms approved by a court. Which constitutional provision provides the strongest argument against the constitutionality of this statute? (A) The Fifteenth Amendment. (B) The Privileges and Immunities Clause of the Fourteenth Amendment. (C) The Thirteenth Amendment (D) The Due Process Clause of the Fifth Amendment.

(C). Pursuant to the Thirteenth Amendment of the United States Constitution, slavery and involuntary servitude, except as a punishment for crime whereof the party shall have been duly convicted, are prohibited within the United States. Here, the statute in the question creates a system of compulsory service based on debt, which is a form of involuntary servitude, even if ostensibly "voluntary." As such, this provision offers the strongest argument against the constitutionality of the statute. The Fifteenth Amendment provides that the right of citizens of the United States to vote shall not be denied or abridged by the United States or by any state on account of race, color, or previous condition of servitude. This scenario did not involve the right to vote, so this answer is incorrect.

QID: 6946 An army fort had been used as a military outpost for more than 100 years. Following the end of the Cold War, the federal government closed the fort and sold half of the property to the state, which used the land to build a university. The population at the school is now double the population that had existed at the base, straining the available water supply. The water source, while located on the land purchased by the state, supplies water to both halves of the property. As a humanitarian gesture in response to global events, the federal government agreed to take in 200 refugees from a war-torn country, and decided to place them in the barracks that remained on the portion of the land which the federal government still owned. The state then brought suit in federal court to prevent the refugee relocation, claiming that there would not be enough water for the existing population. What is the most likely outcome of the suit? (A) The state will prevail, because a state may assert its regulatory authority over the federal government. (B) The state will prevail, because the source of the water is on state property. (C) The state will not prevail, because of the Supremacy Clause. (D) The state will not prevail, because a state may not assert its regulatory authority over a natural resource to be used by the federal government for the benefit of aliens.

(C). The Supremacy Clause prohibits the state form regulating in a manner that conflicts with valid federal law. Congress has plenary (i.e., absolute) power over aliens. The federal government's decision to place refugees falls within the federal power to set conditions under which aliens may enter or remain in the country. The state may not regulate in a manner inconsistent with federal policy on the refugees. As such, the state will not prevail. The state is not absolutely prohibited form regulating a natural resource used by the federal government for the benefit of non-citizens; it is prohibited only from regulating it in a manner inconsistent with federal policy.

QID: 98313 A plaintiff sued a defendant for violation of the Federal Age Discrimination in Employment Act, which created the cause of action that the plaintiff asserted. The parties agreed to settle the case, and entered into a contract entitled "Settlement Agreement and Mutual Release," in which the plaintiff agreed to release all claims against the defendant in exchange for a future payment of $80,000. The plaintiff and the defendant were both citizens of State A. The defendant failed to make the $80,000 payment when it was due, and the plaintiff filed suit for breach of the settlement agreement in federal court. The defendant moved to dismiss the case for lack of federal subject matter jurisdiction. The court denied the motion. Was the court correct to deny the motion to dismiss for lack of federal subject matter jurisdiction? (A) Yes, because more than $75,000 was in controversy. (B) Yes, because the plaintiff's settled claim under the Age Discrimination in Employment Act arose under federal law. (C) No, because the case for breach of the settlement agreement did not arise under federal law. (D) No, because the plaintiff's settled claim under the Age Discrimination in Employment Act should not have been in federal court to begin with.

(C). The plaintiff had a claim that arose under federal law, but released it as part of the settlement. The plaintiff's claim now arises solely from the breach of contract, which is a state law claim.

A state's Occupational Health and Safety Board recently issued regulations valid under its statutory mandate requiring that all employers in the state provide ionizing air purification systems for all employee work areas. These regulations replaced previous guidelines for employee air qualit, which were generally not mandatory and did not specify the method of air purificaiton used. The requirements regarding air puridificaiton systems are least likely to be consittutional as applied to which of the following employers? (A) A company that provides personalized instruction for private citizens in the use of home computers that is a wholly owned Japanese subsidiary of a Japanese corporation. (B) The state Supreme Court, which recently complete construction of its new courthouse complete with non-ionization air purification systems that are serviced under a five-year contract signed prior to enactment of the new regulations. (C) The United States Armed Forces Recruting Center lcoated in the capital city of the state. (D) A privately operated community service center funded by donations and constructed through use of a loan provided by the Veterans Administration and repayable to that agency.

(C). The state would be prohibited from directly regulating the federal government. The Supremacy Clause prohibits state or local governments from directly regulating or taxing the federal government without the federal government's consent. Here, the Armed Forces Recruiting Center is an agency of the federal government, which is not bound by state regulation of its facilities. The fact that many federal buildings do comply with state and local building codes and zoning laws is due to federal consent, not constitutional requirements.

QID: 108929 In a plaintiff's negligence action against a defendant arising out of a multiple-car collision, a witness testified for the plaintiff that the defendant went through a red light. On cross-examination, the defendant sought to question the witness about her deposition testimony given in a separate action between two other drivers who were also involved in the accident. In a pretrial hearing, the judge ruled that the transcript of the deposition was self-authenticating The plaintiff objected to the introduction of the deposition transcript. How should the court rule with regards to the transcript. How should the court rule with regards to the transcript? (A) The transcript is admissible for impeachment only, because the witness is available. (B). The transcript is admissible as substantive evidence only because the witness is available. (C) The transcript is admissible for impeachment and as substantive evidence. (D) The transcript is inadmissible, because it is hearsay not within any exception.

(C). This question asks for what purpose the deposition testimony can be used or admitted. Under FRE, a statement is not hearsay and thus may be used either for impeachment or substantive evidence if it is either (1) inconsistent with the declarant's current testimony and was given under oath at an earlier proceeding; or (2) consistent with the declarant's current testimony and offered to rebut a charge of recent fabrication, to rehabilitate credibility, or to identify someone the declarant perceived earlier. The declarant must be testifying and subject to cross-examination in the current trial. Here, the witness gave a prior inconsistent statement under oath at an earlier deposition. Consequently, the transcript of that deposition is not hearsay and is admissible for either substantive or impeachment purposes.

QID: 105034 A local TV news anchor aired a show in which she stated that the mayor was "a corrupt politician who manages to grab a surprisingly large amount of bribe money with his small hands." The mayor was the subject of a recall effort that was ultimately successful, and he sued the TV station for defamation. The TV station defended on the grounds of truth. The mayor took the stand and testified that he was an honest politician. The station called a witness to testify that a year prior to the TV show in question, the mayor took a bribe in connection with a rezoning project. The mayor's lawyer objects. Should the court allow the testimony to be admitted into evidence? (A) No, because it is inadmissible character evidence. (B) No, because the prosecutor may not introduce evidence of character unless the defendant puts his character in issue. (C) Yes, as relevant evidence of the mayor's character as a corrupt politician. (D) Yes, as relevant evidence of character for untruthfulness.

(C). When a person's character is in issue, character is admissible in all forms. Here the mayor's character as a corrupt (or honest) politician is in issue because it was the subject of the defamatory statement and the defense of truth is asserted.

QID: 108873 The plaintiff was hit by a car while crossing a street late one night. The plaintiff filed a negligence action against the driver of the car. The complaint contained the following allegations: 1) On August 3, 2015, at approximately 11:30 PM, the plaintiff walked to the convenience store across the street from her apartment to buy some ice cream and cereal. After making her purchases, the plaintiff left the convenience store. 2) The entrance to the convenience store is on Ocean Street directly across from the entrance to the plaintiff's apartment building, so the plaintiff did not walk to the corner to cross the street at the light. However, plaintiff looked both ways and confirmed there was no traffic before beginning to cross the street. 3) The defendant was speeding when he turned the corner onto the Ocean Street. Because of the speed at which the defendant was driving he was unable to adequately stop his vehicle when he observed the plaintiff crossing the street. The defendant his plaintiff with his car. 4) The plaintiff suffered several broken bones and internal injuries as a result of the collision. The plaintiff incurred medical expenses and lost time form work as a result of the collision. The plaintiff lives in a jurisdiction that follows common law contributory negligence. The defendant filed a Fed. R. Civ. P. 12(b)(6) motion to dismiss the plaintiff's complaint for failure to state a claim upon which relief can be granted. What is the likely ruling on the defendant's motion to dismiss? (A) The trial court will grant the motion, because common law contributory negligence is a complete bar to recovery. (B) The trial court will grant the motion, because there are insufficient facts to support the plaintiff's negligence claim. (C) The trial court will deny the motion, because the plaintiff's admission that she did not cross the street at the light does not negate any elements of negligence. (D) The trial court will deny the motion, because there are sufficient facts plead in the complaint which, if proven at trial, establish that the defendant is liable for the plaintiff's injuries.

(D). A court will dismiss a complaint for failure to state a claim upon which relief can be granted if the complaint: (1) fails to state a cognizable claim; (2) provides insufficient facts; or (3) contains an allegation that negates one or more elements of the cause of action. Negligence requires establishing duty, breach, causation, and damages. Common law contributory negligence is a complete bar to recovery. Based on all the facts pleaded in the plaintiffs' complaint, there are sufficient facts to establish that: (a) the defendant breached his duty to act with reasonable care by speeding and caused plaintiff's damages by hitting her with his care; and (b) the plaintiff was not contributorily negligence because she exercised reasonable care by looking both ways before crossing the street even if she did not cross at the light.

QID: 7310 The State of Madison has a statute that provides as follows: (1) first-degree murder is a deliberate and premediated killing, (2) second-degree murder is an unlawful killing with malice aforethought, and (3) manslaugher is either an unlawful killing committed with adequate provacation or an unlawful killing committed through criminal negligence. An electrician, a resident of the State of Madison, is deistraught over his recent dismissal from a job he had held for 20 years. The day following his dismissal, the electrician returns to his former employer's place of business with two loaded handguns. The electrician enters the building and confronts the owner of the business. Waving the two handguns around indiscriminately, he shouts, "You'd better give me my job back, or I'm going to start shooting!" Concerned for the safety of his employees and customers, the owner dives at the electrician in an attempt to disarm him. As the owner takes the electrician, one of the handguns hits the ground and discharges. A customer is shot and killed. The owner is guilty of which, if any, of the following crimes? (A) First-degree murder (B) Second-degree murder. (C) Manslaughter. (D) No crime.

(D). A defendant is justified in defending another person with reasonable force only if he reasonably believes the victim had a right to use such force. Some jurisdictions limit this defense to situations where a special relationship exists between the defendant and the victim, while other jurisdictions view the defendant as "standing in the shoes" of the person defended. The owner committed no crime. He was not the aggressor, and his actions were taken in defense of himself and others. He had a reasonable belief in the threat of imminent death or serious bodily harm; he used non-deadly force. Even if there were any possibility that the owner's actions could be construed as negligent, his behavior did not exhibit the heightened degree of deviation from the "reasonableness" standard required to be classified as criminal negligence.

QID: 98332 A man sued a corporation in federal court in State A for injuries the man suffered when the motorcycle the corporation manufactured in State B malfunctioned. Diversity jurisdiction existed. The product was never in State A. The corporation was incorporated in State C and maintained its principal manufacturing facility and its corporate headquarters in State A. The corporation filed a timely motion to dismiss for lack of personal jurisdiction. Should the court grant the motion to dismiss for lack of personal jurisdiction? (A) Yes, because the corporation is not incorporated in State A. (B) Yes, because the claim did not arise form or relate to any contact between the corporation and State A. (C) No, because the corporation's contacts with State A were purposeful. (D) No, because the corporation is essentially at home in State A.

(D). A federal court is going to have general personal jurisdiction over a corporation in two instances: (1) in the state where the corporation is incorporated; and (2) in the state where the corporation is "essentially at home". When the corporation is essentially at home, the court will have general personal jurisdiction over the corporation. Because the corporation's principal manufacturing facility and its corporate headquarters were both in State A, it is likely that the corporation will be deemed as essentially a at home in State A. As such, a State A federal court will have general jurisdiction over the corporation.

QID: 6816 To alleviate the burden on the U.S. Supreme Court from litigation challenging public school achievement testing, Congress enacted jurisdictional legislation that required all secondary school pupils to meet a certain level of competency before being permitted to graduate and removing the Supreme Court's jurisdiction of competency testing cases brought on a racial basis. Advocacy groups claimed that the tests were discriminatory and unlawfully prevented the graduation of students of special racial backgrounds. After enactment of the legislation, a student challenged the Texas state achievement testing in federal court on equal protection grounds, arguing that 64% of Latino students taking the test failed to meet competency levels, while only 23% of non-Latino students did. The federal court found the Texas test nondiscriminatory. The student appealed to the Court of Appeals, which upheld the lower federal court. Finally, the student appealed to the Supreme Court. What is the most likely outcome of the appeal? (A) The Supreme Court will not hear the case, because of Congress' plenary power under Article III. (B) The Supreme Court will not hear the case because doing so would enlarge the Court's jurisdiction under the new federal law. (C) The Supreme Court will hear the case because the Supreme Court has appellate jurisdiction of cases implicating the Equal Protection Clause. (D) The Supreme Court will hear the case, because denial of jurisdiciton by Congress would deny the essential role of the Supreme Court.

(D). Congress cannot remove the appellate jurisdiction of the U.S. Supreme Court in a way that thwarts the essential constitutional role of the Court. The Constitution grants the Supreme Court appellate jurisdiciton "as to law and fact, with such exceptions, and under such regulations as the COngress shall make." The power of Congress to affect Supreme Court appellate jurisdiction is limited to the extent that Congress cannot deny the Court jurisdiction of a class of cases that involve equal protection (or other constitutional) claims, such as the racial claims here. Congressional removal of jurisdiciton in such cases would violate the separation of powers doctrine.

QID: 95009 A teenager sued as the sole representative of a class of person who had bought a weight loss pill over the internet that was ineffective, from a diet supplement company. The teenager commenced a diversity-based class action against the diet supplement company in the appropriate U.S. District Court. Which of the following is true? (A) If the court refuses to certify the proposed class, the teenager has 20 days to appeal. (B) Assuming the class is certified, if the teenager prevails in the suit, she is not entitled to recover attorney's fees unless a federal statute explicitly provides for such an award. (C) Assuming the class is certified, the teenager may enter into a settlement agreement with the diet supplement company, so long as notice is given to all members of the class and a majority approve of the compromise. (D) Assuming the class is certified, any settlement of the class action must be approved by the court.

(D). FRCP requires that a settlement of a class action be approved by the court. A court of appeals may permit an order granting or denying class-action certification under this rule if a petition for permission to appeal is filed with the circuit clerk within 14 days after the order is entered. FRCP allows the award of reasonable attorney's fees and nontaxable costs that are authorized by law or by the parties' agreement. It is not sufficient for a settlement of a class action to be approved by a majority of the class members; such a settlement must be approved by the court.

QID: 90289 A composer sued a film company for copyright infringement in federal court. The initial jury was composed of 12 jurors; however, during the trial one of the jurors became sick and was excused by the judge form further service. This juror was not replaced. After jury deliberations, ten of the remaining jurors voted that the film company had infringed the composer's copyright. One of the jurors abstained form the vote, on the grounds that he was a songwriter and therefore likely to be biased. May the jury render a verdict at this time that the defendant infringed the plaintiff's copyright? (A) Yes, because ten jurors will have participated in the verdict. (B) Yes, because the juror who abstained form the vote has an actual bias. (C) No, because not all of the jurors selected for the trial will have participated in the verdict. (D) No, because not all of the jurors remaining at the end of the trial will have participated in the verdict.

(D). In federal court all jurors must participate int he verdict, unless excused. Here, there is one excused juror, and therefore that juror does not participate in the verdict. However, the juror alleging that he may be biased has not been excused and therefore must participate int he verdict. As such, the jury may not render a verdict at this point.

QID: 8413 A rural community has experienced a building boom in the past decade, causing significant strains on municipal services. With a view to controlling residential and commercial building, and stabilizing the town's population, the community enacts a zoning ordinance requiring a two-acre lot minimum for the construction of single-family dwellings. The community also restricts the number of building permits to be issued annually, totaling no more than 50 residential permits and 10 commercial permits per year. According to a city planner hired by the community to conduct a study, if all allowable permits are issued each year, the community will be build out (that is, no more buildable land will be available) within 15 years. A builders association, headquartered in a nearby town, is an organization of family-owned companies specializing int he construction of affordable housing, including single-family homes and small apartment and condominium developments. If the builders association sues to enjoin enforcement of the two-acre lot minimum size requirement for residential construction, on whose injury must the builders association establish standing? (A) Builders who have pending projects in the community. (B) Family-owned construction companies that would have difficulty asserting their own rights. (C) Home buyers who would like to live in the community. (D) Builders association members who build residential property in the community.

(D). The builders association can assert association standing to assert the rights of its members. Association standing requires that an action be brought by the association on behalf of its members when: 1) the members themselves would have standing; 2) the interests the association seeks to protect are germane to the association's purpose; and 3) neither the claim asserted nor the relief requested requires the members to participate. Here, members of the association would have standing because they are suffering a direct economic injury caused by the two-acre minimum lot size requirement: the members' business of constructing affordable homes is adversely affected economically by the large lot size required to build a single family home. The builders association's injunctive action is germane to the association's purpose to promote the construction of affordable housing. Finally, builders association members need not participate in the builders association action. Thus, association standing exists.

QID: 81023 More than anything else, an elderly widow wanted a seaside cottage. She bought a lot along the ocean and hired a local carpenter to build her cottage. Construction began late in the year. By July, the carpenter had finished about 75% of the work on the cottage when a hurricane hit and completely destroyed the cottage. The carpenter refused to build a new cottage for the widow, and the widow refused to pay the carpenter anything for the work he had performed. The carpenter filed a breach of contract action against the widow. What amount is the carpenter entitled to recover from the widow for his work on the cottage? (A) 75% of the contract price. (B) Either the fair market value of his partial cottage construction at the time of the hurricane or the increase in value to the widow from the work on the cottage before the hurricane. (C) His expected profit from the cottage contract, if he can reasonably establish that value. (D) Nothing, because a condition under the contract was not fulfilled.

(D). In this case, the condition to be fulfilled before payment was the completion of the cottage. The cottage was destroyed by an act of nature, not through any negligence won the widow's part. The occurrence of the destructive force of nature was not unforeseeable--such events happen all the time--but the parties failed to allocate the risk, nor is the carpenter's duty to perform discharged by impossibility. Although it would be very expensive for him to perform, courts are reluctant to call such performance "impossible." As such, the carpenter's duty to build the cottage is not discharged by the destruction of the building; he is required to rebuild, and his refusal to do so constitutes a willful breach of contract. He will be liable to the widow for breach of contract if he does not fulfill her contractual obligations.

QID: 6608 A retired teacher dried and sold a variety of flowers, herbs, and spices. While vacationing, an executive from a major potpourri manufacturer visited the teacher's roadside shop, and impressed with the quality, negotiated a contract with the teacher. The contract provided that the executive "agrees to purchase all of the mountain wildflower crop harvested by the teacher during the next five years at the current market price at time of delivery." For more than 2 years, the teacher delivered to the executive, and the executive paid for all of the mountain wildflowers the teacher harvested. During that time, the teacher purchased more land and hired more employees, thereby doubling her production. However, she soon found that she missed her visits with the tourists who passed through town and the locals who used to stop by her shop. She proposed to the executive that because she had increased production so dramatically it would only be fair that she supply him with only half of her new total, but in any event a minimum of 500 pounds per year, leaving her free to sell the rest to tourists and locals as before. The executive agreed to the proposal for the remaining period of the original contract. Was the original agreement between the teacher and the executive enforceable? (A) No, because there was no definite price. (B) No, because the teacher's promise to sell all her harvest was not supported by any consideration from the executive. (C) Yes, for the first five years, because it is a "reasonable" period of time. (D) Yes, in all respects.

(D). Output contracts (i.e., ones in which the buyer agrees to purchase everything the seller produces) do not involve illusory promises and are enforceable. They are specifically approved by the UCC< which implies a covenant of good faith and precludes unreasonable demands.

QID: 92373 ABC Corporation was incorporated and had its place of business in State A. It made widgets. The tor, a key component of the widget, was made by BCD Company incorporated and with its place of business in State B. ABC sued BCD in federal court for damages from defective tors in the amount of $500,000. During discovery, ABC sent 10 requests for admission to BCD. One stated that BCD made the tors that were used in making the widgets at issue. BCD made the admission. Just before discovery ended, BCD found out that ABC bought tors from a different company in the past. During trial, BCD denied that it made the tors that caused the widgets to break and sought to litigate the issue of who made the tors. May BCD raise the issue at trial? (A) Yes, because ABC previously bought tors from a different company. (B) Yes, because BCD did not know that ABC had bought tors from a different company. (C) No, because BCD did not file a motion to bring in the other ocmpany that had provided tors to ABC. (D) No, because BCD agreed to ABC's request for admission that BCD had made the tors at issue in the case.

(D). Pursuant to FRCP, a matter admitted is conclusively established unless the court, on motion, permits the admission to be withdrawn or amended. Here, BCD did not move to withdraw the admission, and it will be bound by it.

QID: 12931 A retailer sells kitchen appliances. A large corproation (which is the manufacturer) has befun selling a new blender. The retailer has never sold products form this corporation before, but would like to begin carrying this new blender. The corporaiton has a reputation for producing high-quality goods. When the first shipment of blenders arrives at the retailer's store from the corporation, the employees eagerly unpack them and place them right on the shelves. Within a few hours, all of the blenders have been sold. A woman buys one of the blenders and uses it that evening to make smoothies for her friends. However, the blender malfunctions when the blade, which is very loose, flies off and hits the woman in the face. The woman now seeks to sue the retailer for products liability under a negligence theory, because the retailer breached its duty of care owed to the woman. The retailer asserts that it cannot be held liable under this theory because it did not breach its duty of care to the woman. Who is correct? (A) The woman, because the retailer had a duty to inspect goods which were supplied by a new supplier. (B) The woman, because the retailer is in the chain of distribution of the blenders. (C) The retailer, because the retailer does not have to inspect packaged goods for defects unless the retailer received customer complaints regarding the product. (D) The retailer, because failure to inspect packaged goods for defects is not a breach of duty if the goods come from a reputable manufacturer or distributor.

(D). Regarding a products liabilty case based on a negligence theory, in general, a failure to inspect packaged goods for defects is not a breach of duty if they come from a reputable manufacturer or distributor. If the goods are manufactured or otherwise supplied by a previously unknown or questionable source, defendant's unreasonable failure to inspect is a breach of duty. It would, however, probably be a breach of duty not to inspect a particular product as to which defendant had received or become aware of customer complaints. In this case, the retailer purchased the blenders from a manufacturer with a reputation for producing high-quality goods. Therefore, the retailer's failure to inspect the packaged blenders is not a breach of duty, nd this answer choice is correct. Note that the retailer could be held liable on a strict liability theory because it is in the distribution chain, even if it is without fault.

QID: 8488 A student asked to borrow her roommate's sapphire earrings to wear to her sorority formal because they matched her blue dress. The roommate refused. On the night of the formal, the roommate was away visiting her parents, and so the student wore the earrings, intending to return them after the event. Late in the evening, the student removed the earrings because they were pinching her earlobes, and left them on the table in front of a friend. The student asked the friend to "keep an eye on" the earring sand then returned to the dance floor without waiting to hear whether the friend agreed. An hour later, a waiter noticed the earrings on the table. He pawned the earrings for cash but intended to reclaim them and return them to the owner if anyone inquired about lost sapphire earrings. Who is guilty of larceny? (A) The student and the waiter. (B) Only the student. (C) Only the waiter. (D) No one.

(D). The crime of larceny requires a taking and carrying away of the personal property of another with intent to permanently deprive the owner of possession of the property. Here, the owner of the earrings refused the student's request to borrow them, and so the elements of unauthorized taking and carrying away of personal property were met when the student borrowed the earrings and wore them to her formal without the owner's consent. Similarly, thw waiter committed an unauthorized taking and carrying away of another's personal property when he took the earrings form the table and pawned them. However, neither the student nor the waiter had the intent to permanently deprive the owner of the earrings of possession of them. The student intended merely to borrow the earrings for an evening and meant to replace the earrings immediately after the event, in the same condition in which she found them. She meant to deprive the owner of possession only temporarily, and although her care of the earrings was reckless, she did not borrow them with the intention of recklessly abandoning them and hoping someone else would return them. She did not have the requisite intent to commit larceny, which is a specific intent crime. For his part, the waiter intended to reclaim the pawned earrings if someone reported them lost. Although it is a close call, the court may find that because he intended to return the earrings if the owner identified herself, he lacked the requisite intent to commit larceny. Courts have held that a person who pawns an item while waiting for the real owner to claim the item with the intent that, should the real owner do so, he will return the item, is not guilty of larceny. In that sense, it's not too dissimilar to finding a lost item and holding on to it for yourself waiting for the real owner to claim it. While it is indeed a close call, remember that larceny is a specific intent crime (meaning a defendant cannot be guilty unless he or she has the requisite intent) and the intent for larceny is an intent to permanently deprive the owner of the property. Because the waiter did not have that intent here, the waiter will not be guilty of larceny, and so this question is not relying on the whim of a judge, but rather the intent requirement for larceny.

QID: 92767 A defendant is a 40-year-old bachelor still living with his parents. Recently, his parents have informed him that they will begin charging him rent, and will no longer pay his tuition at the local community college. Desperate for money, defendant devised a plan. He convinced his neighbor that his parents had a large amount of money in a safe hidden in their bedroom. The defendant claimed that the money was his life savings that his parents stole from him. He convinced the neighbor to enter the home one night and remove the money from the safe and return it to the defendant. In truth, and unknown to the neighbor, the money belonged to the parents. The neighbor did as the defendant requested, entered the home at night, and removed the money from the safe. The police arrived just as the neighbor left the home. He immediately told his story to the police. Both the defendant and the neighbor were charged with burglary and larceny. With regard to proper convictions, which of the following is most accurate? (A) Both the defendant and the neighbor are guilty of burglary and larceny. (B) The defendant alone is guilty of burglary and larceny. (C) The neighbor alone is guilty of burglary and larceny. (D) The defendant alone is guilty of larceny.

(D). The defendant cannot be guilty of burglary because it requires as one of its elements that the be a breaking and entering of the dwelling of another. This is the defendant's own home. The remaining issue is whether defendant can be guilty of larceny when he did not himself "take and carry away the personal property of another."In other words, although the defendant had the requisite mens rea for larceny, he did not commit the actus reus. Nevertheless, he is guilty of larceny because he employed an "innocent dupe" to engage in the requisite conduct supporting a larceny conviction. The neighbor is guilty of nothing, lacking the required mens rea for larceny or burglary.

QID: 94690 A state statute provides as follows: "In all criminal cases, the burden of proof as to a defense claimed by the defendant shall rest on the defendant, and the magnitude of the burden shall be as great as the Constitution allows." The same state defines the crime of burglary as the "breaking or entering of the dwelling house of another at night with the intent to commit a felony or theft therein. If the house of another belongs in part to the defendant, that shall be a complete defense to the crime of burglary." At a defendant's trial for burglary, he testified that the residence was partially his own, because he had resided there iwth the victim. How should the trial judge instruct the jury regarding the issue of ownership as a defense? (A) The burden of proving ownership, by a preponderance of the evidence, rests on the defendant. (B) The burden of proving ownership, by clear and convincing evidence, rests on the defendant. (C) The burden of proving ownership by proof beyond a reasonable doubt, rests on the defendant. (D) The burden of proving ownership, by proof beyond a reasonable doubt, rests on the prosecution.

(D). The statute above provides that an element of the crime of burglary is the dwelling house "of another." Therefore, because it is an element of the offense, the burden rests with the prosecution to prove it beyond a reasonable doubt. That burden can never shift to the defendant as that would violate the defendant's due process rights.

QID: 8128 A grocery shopper was shopping at the local market when he realized he needed to use a bathroom. He asked an aemployee where the bathroom was located. The employee replied that there was a plumbing problem in the public bathroom and that as a result the bathroom was closed to use. The grocery shopper pleaded with the employee if there was another bathroom he could use. THe employee replied, "Well, there is an employees-only bathroom located in the storage room for the store. I'm not supposed to allow customers into the storage room, but if you are in and out quickly you can use that bathroom." The grocery shopper thanked the employee, received directions to the employee bathroom, and made immediate use of the facilities.As he left the bathroom and was walking through the storage room to return to the market, the owner of the market emerged from his office, which adjoined the storage room. Since the market had experienced a number of thefts from the storage room recently, the storage owner assumed that the grocery shopper was attempting to steal items from the storage room. He grabbed the grocery shopper by the arm and shoved him into the office, locking the door behind so that the grocery shopper could not escape. Moments later, the employee ran into the storage room and explained the situation to the owner. The store owner quickly released the grocery shopper from the office and apologized for the mistake. If the grocery shopper sues the store owner for battery, who will prevail? (A) The store owner, because he was acting under a reasonable belief that he was defending his property. (B) The store owner, because the bathroom was for employees only and not open to the public. (C) The grocery shopper, because he had permission to use the restroom. (D) The grocery shopper, but only if the store owner used force that was unreasonable under the circumstances.

(D). The store owner touched the grocery shopper when he grabbed him by the arm and shoved him into the office. However, under the "shopkeeper's privilege," the store owner had the right to use reasonable force to detain the grocery shopper for a reasonable time to investigate a possible theft. Under the circumstances, the store owner would have reason to believe that the grocery shopper may have been committing a theft. Therefore, the store owner would be privileged to act, provided he used reasonable force. So, the grocery shopper could recover only if the store owner used unreasoanble force in detaining the grocery shopper.

QID: 108853 State X was trying to reduce its spending onpublic assistance benefits. In order to accomplish this goald, state X passed a new law that prohibited its unemployment agency from providing unemployment benefits to anyone who had less than a total of one year employment in State X. A plaintiff worked as a full-time engineer in State Z for over a decade. Six months ago, the plaintiff moved to State X to live closer to family members. The plaintiff was laid off from his job and applied for unemployment benefits in State X. Under the new law, State X's unemployment agency denied the plainitiff's application for unemployment benefits because the plaintiff had only worked in State X for six months. Will the plaintiff be successful in challenging State X's new law on due process grounds? (A) No, because State X has a compelling state interest in reducing its spending. (B) No, because the plaintiff does not have a vested property right in unemployment benefits. (C) Yes, because the new law is arbitrary and unreasonable. (D) Yes, because the new law will not survive strict scrutiny review.

(D). The substantive due process doctrine is used to evaluate governmental regulations that affect fundamental rights of personhood. Strict scrutiny review applies to laws that burden the exercise of fundamental rights. Under the strict scrutiny standard, the burden of persuasion is on the government to prove that the measure being challenged is necessary to further a compelling state interest. Necessary means that there is no less restrictive alternative means available. Denying public benefits based on residency requirements interferes with the fundamental right to travel. Reducing spending is not likely a sufficiently compelling government interest to survive strict scrutiny review.

QID: 31527 A family lived in a quiet residential neighborhood. They built a tree house in the backyard for their three adolescent children. One weekend, the family took a vacation to a neighboring state. While the family was away, a 10 year old youngster who lived in the neighborhood trespassed upon the property. The boy was in the family's backyard when he saw the tree house, which was situated in a tall tree. The boy climbed up the ladder and began playing in the tree house. While inside, the boy peered out the window and saw a large limb with avocados perched above the tree house. The boy decided to collect a few avocados to bring home to his mother. After the boy was unable to reach the avocados, he climbed on top of the tree house roof in an effort to get closer to the avocado branch. As the boy was walking along the roof, he slipped and fell to the ground. This fall caused the boy to severely fracture his ankle and leg. If the boy's parents assert a tort action against the family to recover for the boy's injuries, will the plaintiff's prevail? (A) Yes, if the tree house constituted an attractive nuisance. (B) Yes, if the boy's own negligence was the cause of his injury. (C) No, if a child of like age, intelligence, and experience would not have climbed on top of the roof of the tree house. (D) No, if the boy recognized the risk that was involved.

(D). Under the attractive nuisance doctrine a possessor of land is subject to liability for physical harm to trespassing children where: (1) an artificial dangerous condition exists on the property; (2) the possessor knows or should know that children are likely to trespass on the property; (3) the children because of their youth, age, or immaturity fail to appreciate the danger or realize the risks involved in intermeddling with it; and (4) the utility of maintaining the dangerous condition is slight compared to the risks involved. It is important to note that the inability of a child to appreciate the danger is a subjective test

QID: 12798 A man was up for a promotion at his law firm. His boss was very encouraging as to the man's chances of being made a parter at the firm, but the final decision would be made following an "informal" dinner party with all the firm's partners and their wives at the boss's home. The law firm prided itself on employing legal minds who were more socially astute than typical lawyers, and employees' charisma and people skills were valued equally with intelligence and experieince. In addition, as the partners did a great deal of entertaining of clients, a partner's spouse needed to be just as impressive as the partner himself. In preparation for the big party, the man's wife went to her regular hair salon for a color touch-up and styling. Because her usualy stylist was on vacation, the wife agreed to be seen by a stylist who'd only recently joined the salon. However, the new stylist had little experience coloring hair fo the wife's shade and rtexture, and the result was a disastrous glaring reddish shade with tints of maroon. Appalled, the wife left the salon and went home and washed her hair several times, hoping to dim the effects. She did the best she could to style her hair elegantly for the dinner party, but when she arrived at the boss's home, the dim lighting made the wife's hair look practically purpole. "Trashy" was the whispered consensus of the partners' wives. The partners agreed that the man just wasn't partner material, and the boss advised him that while he was welcome to continue on staff, he was unlikely to progress to a partner. Convinced he would have made partner if not for his wife's stylist's coloring gaffe, the man filed suit against the salon to recover the $100,000 signing bonus he would have received upon making partner as well as the difference in salary and profits. The man stated two causes of action against the salon, one in negligence and one in intentional interference with business relations. Is the man likely to prevail in his negligence claim against the salon? (A) Yes, because the stylist's act was a substantial factor int he man's not receiving the promotion. (B) Yes, if the man can prove actual damages. (C)No, because the man was not a customer of the salon. (D) No, because the man was not a foreseeable plaintiff wihtin the "zone of danger."

(D). Under the majority view, a defendant owes a duty of care only to foreseeable plaintiffs who are within the "zone of danger", that is, those to whom, under the circumstances, a reasonable defendant would have foreseen a risk of harm. Furthermore, the doctrine of proximate cause serves to limit a defendant's liability for unforeseeable damages. Liability is cut off where a court finds highly extraordinary the relationship between the defendant's conduct and the resulting harm. Alternatively, many jurisdictions extend liability only to the foreseeable consequences of a defendant's negligence. Here, under either analysis, it is doubtful that the man's loss of a large promotion and corresponding loss of income would be considered a foreseeable consequence of the salon's botched hair-coloring session with the man's wife.

QID: 10887 A husband and wife owned a large amount of rural farmland. The couple had four children: two sons and two daughters. The three older children all moved away from the farm, but the youngest daughter stayed and continued working on the farm with her parents. The husband and wife executed reciprocal wills which transferred their entire estates to the surviving spouse. After the death of the surviving spouse, the estate would be split equally between the four children. In 2010, the husband died and the wife inherited the estate. Approximately a year later, the couple's youngest daughter purchased her own farm on adjoining farmland. Then the wife transferred the majority of farmland to the daughter's farm. The wife kept the farmhouse she lived in and approximately four acres of property. Because of the transfer of farmland, the wife's estate was devalued by several million dollars. The next year, the wife died. The oldest son sought to void the transfer of farmland to they ounger daughter by arguing undue influence. To support his case, the oldest son moved to introduce evidence establishing that the wife believed her old farmhouse was haunted and that she interacted with the ghosts. The evidence included at least 8 years worth of diaries and videos written and recorded by the wife. Theyoungest daughter appears in several of the videos discussing the potential ghosts with the wife. The youngest daughter objects to the evidence. What is the youngest daughter's best argument to exclude the evidence? (A) The evidence is irrelevant. (B) The evidence is unfairly prejudicial to the wife. (C) The evidence is confusing and misleading. (D) The evidence is a waste of time and needlessly cumulative.

(D). evidence is relevant if: (1) it has any tendency to make a fact more or less probable than it would be without the evidence; and (2) the fact is of consequence in determining the action. Under FRE 403, the court may exclude relevant evidence if its probative value is substantially outweighed by a danger of one or more of the following: (1) unfair prejudice; (2) confusing the issues; (3) misleading the jury; (4) undue delay; (5) wasting time; or (6) needlessly presenting cumulative evidence. Rule 403 is commonly referred to as the balancing test because it requires the judge to balance competing interests of probative value and other concerns. The key analytical focus when dealing with this test is the language "substantially outweighed by a danger of." Rule 403 will thus only exclude that evidence that is significantly more prejudicial than probative, and it sets a low bar for admissibility. First, the evidence is relevant to the son's undue influence claim. An aggrieved party may avail himself of the defense of undue influence when the circumstances reveal a vulnerable, but not incapacitated party who succumbs to untoward bargaining tactics and pressures from the other party, where those pressures or tactics fall short of fraud or duress. There are two elements of undue influence: unfair persuasion was used, and the other party was vulnerable to such persuasion. The wife's belief in ghosts, and the daughter participating in and encouraging the wife's belief make it more probable that the wife was vulnerable to unfair persuasion. However, eight years' worth of diaries and videos is unnecessary to establish this fact. Therefore, the best argument is that all of the evidence is needlessly cumulative and a waste of time.


Kaugnay na mga set ng pag-aaral

Chapter 27 - Soft Tissue Injuries

View Set

Music of Multicultural America midterm

View Set

Chapter 5: Native Americans (W choices)

View Set

Louisiana Life and Health Simulator

View Set

arteriosclerosis and atherosclerosis

View Set

Medical Assistant Pharmacology Pre-Test

View Set